Sunteți pe pagina 1din 39

CO N F L I C T O F L AW S

Nationality & Domiciliary Theory |1


VIII. NATIONALITY THEORY

Our Civil Code (Art. 15) adheres to the theory that jurisdiction
over the status of a natural person is determined by the
latters nationality. Pursuant to this theory, we have jurisdiction
over the status of Baby Rose, she being a citizen of the
Philippines, but not over the status of the petitioners, who are
foreigners.

Topic: Nationality Theory


GR. No. L-16922. April 30, 1963.
IN RE: ADOPTION OF CHILD BAPTIZED UNDER THE NAME
OF
ROSE,
MARVIN
G.
ELLIS
and
GLORIA
C.
ELLIS, Petitioners-Appellees,
v.
REPUBLIC
OF
THE
PHILIPPINES, Oppositor-Appellant.
FACTS: Petitioners are Spouses Marvin G. Ellis, a native of San
Francisco, California, and Gloria C. Ellis in Banger. Both are
citizens of the United States.
Baby Rose was born on September 26, 1959 at the Caloocan
Maternity Hospital. Four or five days later, the mother of Rose
left her with the Heart of Mary Villa stating that she could not
take care of rose.
Being without issue, on November 22, 1959, Mr. and Mrs. Ellis
filed a petition with the Court of First Instance of Pampanga for
the adoption of the aforementioned baby.
At the time of the hearing of the petition on January 14, 1960:
1.

petitioner Marvin G. Ellis and his wife had been in


the Philippines for three (3) years,

2.

he being assigned thereto as staff sergeant in the


United States Air Force Base, in Angeles, Pampanga,
where both lived at that time.

3.

They had been in the Philippines before, or, to be


exact, in 1953.

ISSUE: whether not being permanent residents in


Philippines, petitioners are qualified to adopt Baby Rose.

the

HELD: NO. Article 335 of the Civil Code of the Philippines,


provides that
"The following cannot adopt:
x
x
x
"(4) Non-resident aliens;"
This legal provision is too clear to require interpretation. No
matter how much we may sympathize with the plight of Baby
Rose and with the good intentions of petitioners herein, the law
leaves us no choice but to apply its explicit terms, which
unqualifiedly deny to petitioners the power to adopt anybody in
the Philippines.
ADOPTION PROCEEDINGS ARE IN REM PROCEEDINGS
In this connection, it should be noted that this is a proceedings
in rem, which no court may entertain, unless it has jurisdiction,
not only over the subject matter of the case and over the
parties, but, also, over the res, which is the personal status of
Baby Rose as well as that of petitioners herein.
JURISDICTION OVER THE STATUS OF BABY ROSE ONLY, BUT NOT
OVER PETITIONERS

Under our political law, which is patterned after the AngloAmerican legal system, we have, likewise, adopted the latters
view to the effect that personal status, in general, is determined
by
and/or
subject
to
the
jurisdiction
of
the domiciliary law(Restatement of the Law of Conflict of Laws,
p. 86; The Conflict of Laws by Beale, Vol. I, p. 305, Vol. II, pp.
713-714).
This, perhaps, is the reason why our Civil Code does not permit
adoption by non-resident aliens, and we have consistently
refused to recognize the validity of foreign decrees of divorce
regardless of the grounds upon which the same are based
involving citizens of the Philippines who are not bona fide
residents of the forum, even when our laws authorized absolute
divorce in the Philippines (Ramirez v. Gmur, 42 Phil. 855;
Gonayeb v. Hashim, 30 Phil. 22; Cousine Hix v. Fleumer, 55 Phil.
851; Barretto Gonzalez v. Gonzales, 58 Phil. 67; Recto v. Harden,
L-6897, Nov. 29, 1955)."
Inasmuch as petitioners herein are not domiciled in the
Philippines and, hence, non-resident aliens we cannot
assume and exercise jurisdiction over their status, under either
the nationality theory or the domiciliary theory.
Topic: Determination of Nationality
G.R. No. L-24530
October 31, 1968
BOARD
OF
IMMIGRATION
COMMISSIONERS
and
COMMISSIONER
OF
IMMIGRATION,
petitioners,
vs.
BEATO GO CALLANO, MANUEL GO CALLANO, GONZALO GO
CALLANO, JULIO GO CALLANO and THE COURT OF
APPEALS,
respondents.
The Board of Immigration Commissioners, exercising its power
of review under Commonwealth Act No. 613, issued, also
without any previous notice and hearing, an order reversing the
decision of the Board of Special Inquiry admitting Beato and his
three brothers for entry as citizens; ordering their exclusion as
aliens not properly documented for admission pursuant to
Section 27 (a) (17) of the Philippine Immigration Act of 1940,
and ordering that they be returned to the port whence they
came or to the country of which they were nationals, upon the
ground that they had been able "to enter this country and gain
admission as Filipino citizens by the fraudulently secured
authorization." On the same date, the Commissioner of
Immigration issued a warrant of exclusion commanding the
deportation officer "to carry out the exclusion of the abovenamed applicants (the Go Callano brothers) on the first available
transportation and on the same class of accommodation in
which they arrived to the port whence they came or to the
country of which they are nationals."
The parties ordered deported filed in the Court of First Instance
of Manila an action for injunction to restrain the Board of
Immigration Commissioners and the Commissioner of
Immigration from executing the order of exclusion or
deportation already mentioned. They based their action on the
following grounds: (1) that the Board had no jurisdiction to
exclude them from the Philippines because they were not aliens
but Filipino citizens. Months later, the Court of First Instance
issued a writ of preliminary injunction restraining the

CO N F L I C T O F L AW S
Nationality & Domiciliary Theory |2
respondents in the case from deporting the petitioners. After
trial, the Court rendered judgment finding that, according to
petitioners' undisputed evidence, "the petitioners herein are the
illegitimate children of Emilia Callano, a Filipino citizen, with her
common-law husband a Chinese citizen," and concluding that
"until the petitioners left for China in 1947, they must be
considered as citizens of the Philippines as they were born of a
Filipino mother and an alien father who, however, was not
married to their mother."

with Article 15 of the Civil Code which provides as follows: "Laws


relating to family rights and duties, or to the status, conditions
and legal capacity of persons are binding upon citizens of the
Philippines, even though living abroad." Under Article IV, Section
2, of the Philippine Constitution, "Philippine citizenship may be
lost or reacquired in the manner provided by law," which implies
that the question of whether a Filipino has lost his Philippine
citizenship shall be determined by no other than the Philippine
law.

Notwithstanding the above finding and conclusion, however, the


Court dismissed the case holding that "the petitioners are
citizens of the Republic of China and not being properly
documented for entry into the Philippines as found by the
Immigration Commissioner, the writ of preliminary injunction
heretofore issued by this Court shall be deemed dissolved upon
finality of this decision." The grounds upon which the Court
based its decision were: (1) because petitioners stayed in China
for a period of fifteen years before returning to the Philippines,
they must be considered as citizens of the Chinese Republic; (2)
as petitioners were recognized by their alien father as his
children, they became Chinese citizens under the Chinese law of
nationality. While the Court also found that the cable
authorization mentioned heretofore was a forgery, it held that,
for the purpose of the petition before it, "it was immaterial to
determine the genuineness or falsity of the cable authorization.
For if the petitioners are Filipino citizens, they are entitled to
remain within the territorial jurisdiction of the Republic in
whatever way they might have entered."

Section 1 of Commonwealth Act No. 63, as amended by Republic


Act No. 106, provides that a Filipino citizen may lose his
citizenship by naturalization in a foreign country; express
renunciation of citizenship; subscribing to an oath of allegiance
to support the constitution or laws of a foreign country;
rendering service to, or accepting a commission in, the armed
forces of a foreign country; cancellation of the certificate of
naturalization; declaration by competent authority that he is a
deserter of the Philippine armed forces in time of war; in the
case of a woman by marriage to a foreigner if, by virtue of laws
in force in her husband's country, she acquires his nationality.
Recognition of the petitioners by their alien father is not among
the ground for losing Philippine citizenship under Philippine law,
and it cannot be said that the petitioners lost their former status
by reason of such recognition. About the only mode of losing
Philippine citizenship which closely bears on the petitioners is
renunciation. But even renunciation cannot be cited in support
of the conclusion that petition lost their Philippine citizenship
because the law requires an express renunciation which means
a renunciation that is made known distinctly and explicitly and
not left to inference or implication; a renunciation manifested by
direct and appropriate language, as distinguished from that
which is inferred from conduct. Indeed, as the Supreme Court
held in U.S. v. Ong Tianse, 29 Phil. 332, a case for deportation,
where Ong, a natural child of a Filipino mother and a Chinese
father, born in the Philippines, was brought by his parents to
China when he was 4 years old, where he remained for 18 or 19
years, returning to the Philippines at 25 years of age, "The fact
that a minor child in those conditions was taken to China and
remained there for several years is not sufficient ground upon
which to hold that he has changed his nationality, when, after
reaching his majority, he did not express his desire to choose
the nationality of his father." The import of the foregoing
pronouncement is that of itself a protracted stay in a foreign
country does not amount to renunciation. Moreover, herein
petitioners were all minors when they where brought to China in
1446. They were without legal capacity to renounce their status.
Upon their return to the Philippines only Beato Go Callano had
attained the age of majority, but even as to him there could not
have been renunciation because he did not manifest by direct
and appropriate language that he was disclaiming Philippine
citizenship. On the contrary, after he has attained the age of
majority, he applied for registration as a Philippine citizen and
sought entry into this country, which are clear indicia of his
intent to continue his former status. The foregoing shows that
the petitioners have not lost their Philippine citizenship.

After the denial of herein respondents' motion for reconsideration, they appealed to the Court of Appeals where they
raised the following issues: (a) that being Filipino citizens by
birth, they did not lose their citizenship nor acquire Chinese
citizenship, neither by their prolonged stay in China nor by their
alleged recognition by their Chinese father, and (b) that the
cablegram authorization was not a forgery.
In due time the Court of Appeals rendered the decision now
under review by certiorari, reversing that of the lower court.
In this appeal, the Board of Immigration Commissioners and the
Commissioner
of
Immigration
maintain
the
following
propositions that, granting that they were Filipino citizens when
they left the Philippines in 1946, they lost that citizenship,
firstly, by staying in China for a period of fifteen years, and
secondly, because they were recognized by their common-law
father, they became citizens of the Republic of China in
accordance with the Chinese Nationality Law.
ISSUE: Whether or not petitioners are Filipino citizens? YES
The question, whether petitioners who are admittedly Filipino
citizens at birth subsequently acquired Chinese citizenship
under the Chinese Law of Nationality by reason of recognition or
a prolonged stay in China, is a fit subject for the Chinese law
and the Chinese court to determine, which cannot be resolved
by a Philippine court without encroaching on the legal system of
China. For, the settled rule of international law, affirmed by the
Hague Convention on Conflict of Nationality Laws of April 12,
1930 and by the International Court of Justice, is that "Any
question as to whether a person possesses the nationality of a
particular state should be determined in accordance with laws of
that state. There was no necessity of deciding that question
because so far as concerns the petitioners' status, the only
question in this proceeding is: Did the petitioners lose their
Philippine citizenship upon the performance of certain acts or
the happening of certain events in China? In deciding this
question no foreign law can be applied. The petitioners are
admittedly Filipino citizens at birth, and their status must be
governed by Philippine law wherever they may be, in conformity

As to the validity of cablegram (not important in case lang iask)


Due, therefore, to the pronouncement made by the Court of
Appeals regarding the insufficiency of the evidence presented
by herein petitioners to prove the alleged forgery again, a
matter not now within our power to review the questioned
cablegram must be deemed to be authentic. But be that as it
may, we agree with both the Court of First Instance of origin and
the Court of Appeals that, even assuming that said document
was forged, this would not automatically render void all the
proceedings had before the Philippine Consulate in Hongkong
and the Board of Special Inquiry, both of which ended with a
definite finding that the Callanos were Filipino citizens. That
these proceedings and finding can not be nullified by the
Department of Foreign Affairs summarily and without giving the

CO N F L I C T O F L AW S
Nationality & Domiciliary Theory |3
parties concerned an opportunity to be heard is too evident to
require any demonstration.
Life story nila (not important) were the illegitimate children of
Go Chiao Lin, a Chinese citizen, and Emilia Callano, a Filipino
citizen, who started living maritally in Malitbog, Leyte, in 1934;
that out of their illegitimate union were born the following:
Beato, in Sugod, Leyte, on September 28, 1936; Manuel, in
Libagon, Leyte, on June 17, 1941; Gonzalo, in Malitbog, Leyte,
on April 17, 1943, and Julio in Malitbog, Leyte, on January 31,
1945. The Court of Appeals also found that in 1946, Go Chiao
Lin, Emilia and their four sons went to Amoy, China, on vacation,
but Go died there the same year. In 1948, Emilia had to return to
the Philippines as the maid of Consul Eutiquio Sta. Romana
because she was penniless, leaving her children behind.
Subsequently the latter were able to go to Hongkong, where
they sought and obtained employment. In 1961, they applied
with the Philippine Consul General in Hongkong for entry into
the Philippines as Filipino citizens.
Topic: Dual or Multiple Nationality
GR. No. 83820 May 25, 1990
JOSE B. AZNAR (as Provincial Chairman of PDP Laban in
Cebu), petitioner,
vs.
COMMISSION ON ELECTIONS and EMILIO MARIO RENNER
OSMEA,respondents.

1.

that he is the legitimate child of Dr. Emilio D.


Osmea, a Filipino and son of the late President
Sergio Osmea, Sr.;

2.

that he is a holder of a valid and subsisting


Philippine Passport No.0855103 issued on March 25,
1987;

3.

that he has been continuously residing in the


Philippines since birth and has not gone out of the
country for more than six months;

4.

and that he has been a registered voter in the


Philippines since 1965. (pp. 107-108, Rollo)

ISSUE: what is the citizenship of osmea?


Petitioner's contention that private respondent is not a Filipino
citizen and, therefore, disqualified from running for and being
elected to the office of Provincial Governor of Cebu, is not
supported by substantial and convincing evidence.
In the proceedings before the COMELEC, the petitioner failed to
present direct proof that private respondent had lost his Filipino
citizenship by any of the modes provided for under C.A. No. 63.
Among others, these are:
(1) by naturalization in a foreign country;

PARAS, J.:
The facts of the case are briefly as follows:

(2) by express renunciation of citizenship; and

Private respondent Emilio "Lito" Osmea filed his certificate of


candidacy with the COMELEC for the position of Provincial
Governor of Cebu Province in 1988 local elections.

(3) by subscribing to an oath of allegiance to support the


Constitution or laws of a foreign country.

Petitioner Jose B. Aznar in his capacity as its incumbent


Provincial Chairman, filed with the COMELEC a petition for the
disqualification of private respondent on the ground that he is
allegedly not a Filipino citizen, being a citizen of the United
States of America.
At the hearing before the COMELEC (First Division), the
petitioner presented the following exhibits tending to show that
private respondent is an American citizen:
1.

Application for Alien Registration Form No. 1 of the


Bureau of Immigration signed by private respondent
dated November 21, 1979 (Exh. "B");

2.

Alien Certificate of Registration No. 015356 in the


name of private respondent dated November 21,
1979 (Exh. "C");

3.

Permit to Re-enter the Philippines dated November


21, 1979 (Exh. "D");

4.

Immigration Certificate of Clearance dated January


3, 1980 (Exh. "E"). (pp. 117-118, Rollo)

Private respondent, on the other hand, maintained that he is a


Filipino citizen, alleging:

From the evidence, it is clear that private respondent Osmea


did not lose his Philippine citizenship by any of the three
mentioned hereinabove or by any other mode of losing
Philippine citizenship.
In concluding that private respondent had been naturalized as a
citizen of the United States of America, the petitioner merely
relied on the fact that private respondent was issued alien
certificate of registration and was given clearance and permit to
re-enter the Philippines by the Commission on Immigration and
Deportation. Petitioner assumed that because of the foregoing,
the respondent is an American and "being an American", private
respondent "must have taken and sworn to the Oath of
Allegiance required by the U.S. Naturalization Laws." (p. 81,
Rollo)
Philippine courts are only allowed to determine who are Filipino
citizens and who are not.
Philippine courts are only allowed to determine who are Filipino
citizens and who are not. Whether or not a person is considered
an American under the laws of the United States does not
concern Us here.
By virtue of his being the son of a Filipino father, the
presumption that private respondent is a Filipino remains. It was

CO N F L I C T O F L AW S
Nationality & Domiciliary Theory |4
incumbent upon the petitioner to prove that private respondent
had lost his Philippine citizenship. As earlier stated, however,
the petitioner failed to positively establish this fact.

University Y, presents a Certification that he is a student of


University X, this does not necessarily mean that he is not still a
student of University Y.

The cases of Juan Gallanosa Frivaldo v. COMELEC et al, (G.R. No.


87193, June 21, 1989) and Ramon L. Labo v. COMELEC et
al (G.R. No. 86564, August 1, 1989) are not applicable to the
case at bar.

In the case of Osmea, the Certification that he is an American


does not mean that he is not still a Filipino, possessed as he is,
of both nationalities or citizenships. Indeed, there is no express
renunciation here of Philippine citizenship; truth to tell, there is
even no implied renunciation of said citizenship. When We
consider that the renunciation needed to lose Philippine
citizenship must be "express", it stands to reason that there can
be no such loss of Philippine 'citizenship when there is no
renunciation either "'express" or "implied".

In the Frivaldo case, evidence shows that he was naturalized as


a citizen of the United States in 1983 per certification from the
United States District Court, Northern District of California, as
duly authenticated by Vice Consul Amado P. Cortez of the
Philippine Consulate General in San Francisco, California, U.S.A.
Likewise, in the case of Labo, records show that Labo was
married to an Australian citizen and that he was naturalized as
an Australian citizen in 1976, per certification from the
Australian Government through its Consul in the Philippines. This
was later affirmed by the Department of Foreign Affairs.
In declaring both Frivaldo and Labo not citizens of the
Philippines, therefore, disqualified from serving as Governor of
the Province of Sorsogon and Mayor of Baguio City, respectively,
the Court considered the fact that by their own admissions, they
are indubitably aliens, no longer owing any allegiance to the
Republic of the Philippines since they have sworn their total
allegiance to a foreign state.
Private respondent remains a Filipino and the loss of his
Philippine citizenship cannot be presumed.
In the instant case, private respondent vehemently denies
having taken the oath of allegiance of the United States (p. 81,
Rollo). He is a holder of a valid and subsisting Philippine
passport and has continuously participated in the electoral
process in this country since 1963 up to the present, both as a
voter and as a candidate (pp. 107-108, Rollo). Thus, private
respondent remains a Filipino and the loss of his Philippine
citizenship cannot be presumed.
Certification that he is an American does not mean that he is not
still a Filipino, possessed as he is, of both nationalities or
citizenships
In the learned dissent of Mr. Justice Teodoro Padilla, he stresses
the fact that because Osmea obtained Certificates of Alien
Registration as an American citizen, the first in 1958 when he
was 24 years old and the second in 1979, he, Osmea should be
regarded as having expressly renounced Philippine citizenship.
To Our mind, this is a case of non sequitur (It does not follow).
Considering the fact that admittedly Osmea was both a Filipino
and an American, the mere fact that he has a Certificate stating
he is an American does not mean that he is not still a Filipino.
Thus, by way of analogy, if a person who has two brothers
named Jose and Mario states or certifies that he has a brother
named Jose, this does not mean that he does not have a brother
named Mario; or if a person is enrolled as student
simultaneously in two universities, namely University X and

Parenthetically, the statement in the 1987 Constitution that


"dual allegiance of citizens is inimical to the national interest
and shall be dealt with by law"(Art. IV, Sec. 5) has no retroactive
effect. And while it is true that even before the 1987
Constitution, Our country had already frowned upon the concept
of dual citizenship or allegiance, the fact is it actually existed. Be
it noted further that under the aforecited proviso, the effect of
such dual citizenship or allegiance shall be dealt with by a future
law. Said law has not yet been enacted.
G.R. No. 160869

May 11, 2007

AASJS (ADVOCATES AND ADHERENTS OF SOCIAL JUSTICE


FOR SCHOOL TEACHERS AND ALLIED WORKERS) MEMBER
HECTOR
GUMANGAN
CALILUNG, Petitioner,
vs.
THE HONORABLE SIMEON DATUMANONG, in his official
capacity as the Secretary of Justice,Respondent.
(This case is about the constitutionality of RA 9225 and whether
it violates the Constitutional provision against dual allegiance.)
Facts: Petitioner filed the instant petition against respondent,
then Secretary of Justice Simeon Datumanong, the official
tasked to implement laws governing citizenship. 1 Petitioner
prays that a writ of prohibition be issued to stop respondent
from implementing Republic Act No. 9225, entitled "An Act
Making the Citizenship of Philippine Citizens Who Acquire
Foreign Citizenship Permanent, Amending for the Purpose
Commonwealth Act No. 63, As Amended, and for Other
Purposes."
Petitioners Contention
Petitioner avers that Rep. Act No. 9225 is unconstitutional as it
violates Section 5, Article IV of the 1987 Constitution that states,
"Dual allegiance of citizens is inimical to the national interest
and shall be dealt with by law."
Petitioner contends that Rep. Act No. 9225 cheapens Philippine
citizenship. He avers that Sections 2 and 3 of Rep. Act No. 9225,
together, allow dual allegiance and not dual citizenship.
Petitioner maintains that Section 2 allows all Filipinos, either
natural-born or naturalized, who become foreign citizens, to
retain their Philippine citizenship without losing their foreign
citizenship. Section 3 permits dual allegiance because said law
allows natural-born citizens of the Philippines to regain their
Philippine citizenship by simply taking an oath of allegiance
without forfeiting their foreign allegiance. 2 The Constitution,
however, is categorical that dual allegiance is inimical to the
national interest.

CO N F L I C T O F L AW S
Nationality & Domiciliary Theory |5
SEC. 2. Declaration of Policy.-It is hereby declared the
policy of the State that all Philippine citizens who
become citizens of another country shall be deemed
not to have lost their Philippine citizenship under the
conditions of this Act.
SEC. 3. Retention of Philippine Citizenship.-Any
provision of law to the contrary notwithstanding,
natural-born citizens of the Philippines who have lost
their Philippine citizenship by reason of their
naturalization as citizens of a foreign country are
hereby deemed to have reacquired Philippine
citizenship upon taking the following oath of allegiance
to the Republic:
"I ___________________________, solemnly swear (or
affirm) that I will support and defend the Constitution of
the Republic of the Philippines and obey the laws and
legal orders promulgated by the duly constituted
authorities of the Philippines; and I hereby declare that
I recognize and accept the supreme authority of the
Philippines and will maintain true faith and allegiance
thereto; and that I impose this obligation upon myself
voluntarily without mental reservation or purpose of
evasion."
Natural-born citizens of the Philippines who, after the
effectivity of this Act, become citizens of a foreign
country shall retain their Philippine citizenship upon
taking the aforesaid oath.

confronting the issue of whether or not there is dual allegiance


to the concerned foreign country. What happens to the other
citizenship was not made a concern of Rep. Act No. 9225.
(2) Does this Court have jurisdiction to pass upon the issue of
dual allegiance? NO.
Petitioner: Although Congress has not yet passed any
law on the matter of dual allegiance, such absence of a law
should not be justification why this Court could not rule on the
issue. While it is true that there is no enabling law yet on dual
allegiance, the Supreme Court, through
Mercado v.
Manzano,6 already had drawn up the guidelines on how to
distinguish dual allegiance from dual citizenship. 7
OSG: Pursuant to Section 5, Article IV of the 1987
Constitution, dual allegiance shall be dealt with by law. Thus,
until a law on dual allegiance is enacted by Congress, the
Supreme Court is without any jurisdiction to entertain issues
regarding dual allegiance.
SC: To begin with, Section 5, Article IV of the
Constitution is a declaration of a policy and it is not a selfexecuting provision. The legislature still has to enact the law on
dual allegiance. In Sections 2 and 3 of Rep. Act No. 9225, the
framers were not concerned with dual citizenship per se, but
with the status of naturalized citizens who maintain their
allegiance to their countries of origin even after their
naturalization.9 Congress was given a mandate to draft a law
that would set specific parameters of what really constitutes
dual allegiance.10 Until this is done, it would be premature for
the judicial department, including this Court, to rule on issues
pertaining to dual allegiance.

Defense of OSG
The Office of the Solicitor General (OSG) claims that Section 2
merely declares as a state policy that "Philippine citizens who
become citizens of another country shall be deemed not to have
lost their Philippine citizenship." The OSG further claims that the
oath in Section 3 does not allow dual allegiance since the oath
taken by the former Filipino citizen is an effective renunciation
and repudiation of his foreign citizenship. The fact that the
applicant taking the oath recognizes and accepts the supreme
authority of the Philippines is an unmistakable and categorical
affirmation of his undivided loyalty to the Republic. 3
Issues and Rulings:
(1) Is Rep. Act No. 9225 unconstitutional? NO.
From the excerpts of the legislative record, it is clear that
the intent of the legislature in drafting Rep. Act No. 9225 is to do
away with the provision in Commonwealth Act No. 63 5 which
takes away Philippine citizenship from natural-born Filipinos who
become naturalized citizens of other countries.

Neither can we subscribe to the proposition of


petitioner that a law is not needed since the case of Mercado
had already set the guidelines for determining dual allegiance.
Petitioner misreads Mercado. That case did not set the
parameters of what constitutes dual allegiance but merely made
a distinction between dual allegiance and dual citizenship.
Moreover, in Estrada v. Sandiganbayan,11 we said that the courts
must assume that the legislature is ever conscious of the
borders and edges of its plenary powers, and passed laws with
full knowledge of the facts and for the purpose of promoting
what is right and advancing the welfare of the majority. Hence,
in determining whether the acts of the legislature are in tune
with the fundamental law, we must proceed with judicial
restraint and act with caution and forbearance. 12 The doctrine of
separation of powers demands no less. We cannot arrogate the
duty of setting the parameters of what constitutes dual
allegiance when the Constitution itself has clearly delegated the
duty of determining what acts constitute dual allegiance for
study and legislation by Congress.
Topic: Statelessness

What Rep. Act No. 9225 does is allow dual citizenship to


natural-born Filipino citizens who have lost Philippine citizenship
by reason of their naturalization as citizens of a foreign country.

G.R. No. L-1812

August 27, 1948

On its face, it does not recognize dual allegiance. By


swearing to the supreme authority of the Republic, the person
implicitly renounces his foreign citizenship.

EREMES
KOOKOORITCHKIN, petitioner,
vs.
THE SOLICITOR GENERAL, oppositor.

Plainly, from Section 3, Rep. Act No. 9225 stayed clear out
of the problem of dual allegiance and shifted the burden of

(This case is about an alleged stateless person born in Russia


applying to be naturalized as Filipino.)

CO N F L I C T O F L AW S
Nationality & Domiciliary Theory |6
Facts: Eremes Kookooritchkin applies for Philippine citizenship
naturalization under the provisions of Commonwealth Act 473,
as amended by Act 535.
It was established at the hearing that the petitioner is a nativeborn Russian, having first seen the light of day on November 4,
1897 in the old City of St. Petersburg, Russia.
He grew up as a citizen of the defunct Imperial Russian
Government under the Czars.
(World War I found him in the military service of this
Government. In 1915 he volunteered for the Imperial Russian
navy and was sent to the Navy Aviation School. He fought with
the Allies in the Baltic Sea, was later transferred to the eastern
front in Poland, and much later was sent as a navy flier to Asia
Minor. In the latter part of the war, but before the Russian
capitulation, he was transferred to the British Air Force under
which he served for fourteen months.)
When the revolution broke out in Russia in 1917, he
joined the White Russian Army at Vladivostok and fought
against the Bolsheviks until 1922 when the White
Russian Army was overwhelmed by the Bolsheviks.
As he refused to join the Bolshevik regime, he fled by sea
from Vladivostok to Shanghai and from this Chinese port
he found his way to Manila, arriving at this port as a
member of a group of White Russians under Admiral
Stark in March, 1923.
He stayed in Manila for about seven months, then moved to
Olongapo, Zambales, where he resided for about a year, and
from this place he went to Iriga, Camarines Sur, where he
established his permanent residence since May, 1925. He has
remained a resident of this municipality, except for a brief
period from 1942 to July, 1945, when by reason of his
underground activities he roamed mountains of Caramoan as a
guerrilla officer. After liberation he returned to Iriga where again
he resides up to the present time.
The applicant is married to a Filipino by the name of Concepcion
Segovia, with whom he has one son named Ronald
Kookooritchkin.
Although he could have lived in ease by maintaining good
relations with the enemy by reason of his being Russian-born
during the years preceding the declaration of war by Russia
against Japan, the applicant of his own volition chose to cast his
lot with the guerrilla movement and fought the enemy in several
encounters in the Province of Camarines Sur. He belonged to the
guerrilla outfit of Colonel Padua with rank of major. Upon the
arrival of the forces of liberation he was attached to the
American Army from April to June, 1945.
Although a Russian by birth he is not a citizen of Soviet
Russia. He disclaims allegiance to the present
Communist Government of Russia. He is, therefore, a
stateless refugee in this country, belonging to no State,
much less to the present Government of the land of his
birth to which he is uncompromisingly opposed.
Issue: Whether petitioner is a Russian citizen or is stateless.
--STATELESS

Held:
Appellant contends that the lower court erred in finding
appellee stateless and not a Russian citizen and in not finding
that he has failed to establish that he is not disqualified for
Philippine citizenship under section 4 (h) of the Revised
Naturalization Law.
It is contended that petitioner failed to show that under the laws
of Russia, appellee has lost his Russian citizenship and failed to
show that Russia grants to Filipinos the right to become a
naturalized citizens or subjects thereof. The controversy centers
on the question as to whether petitioner is a Russian citizen or is
stateless.
Petitioner testified categorically that he is not a Russian citizen
and that he has no citizenship. His testimony supports the lower
court's pronouncement that petitioner is a stateless refugee in
this country.
Appellant points out that petitioner stated in his petition for
naturalization that he is citizen or subject of the Empire of
Russia, but the Empire of Russia has ceased to exist since the
Czars were overthrown in 1917 by the Bolshevists, and the
petitioner disclaims allegiance or connection with the Soviet
Government established after the overthrow of the Czarist
Government.
We do not believe that the lower court erred in pronouncing
appellee stateless. Appellee's testimony, besides being
uncontradicted, is supported by the well-known fact that the
ruthlessness of modern dictatorship has scattered throughout
the world a large number of stateless refugees or displaced
persons, without country and without flag. The tyrannical
intolerance of said dictatorships toward all opposition induced
them to resort to beastly oppression, concentration camps and
blood purges, and it is only natural that the not-so-fortunate
ones who were able to escape to foreign countries should feel
the loss of all bonds of attachment to the hells which were
formerly their fatherland's. Petitioner belongs to that group of
stateless refugees.
Knowing, as all cultured persons all over the world ought to
know, the history, nature and character of the Soviet
dictatorship, presently the greatest menace to humanity and
civilization, it would be technically fastidious to require further
evidence of petitioner's claim that he is stateless than his
testimony that he owes no allegiance to the Russian Communist
Government and, is because he has been at war with it, he fled
from Russia to permanently reside in the Philippines. After
finding in this country economic security in a remunerative job,
establishing a family by marrying a Filipina with whom he has a
son, and enjoying for 25 years the freedoms and blessings of our
democratic way of life, and after showing his resolution to retain
the happiness he found in our political system to the extent of
refusing to claim Russian citizenship even to secure his release
from the Japanese and of casting his lot with that of our people
by joining the fortunes and misfortunes of our guerrillas, it would
be beyond comprehension to support that the petitioner could
feel any bond of attachment to the Soviet dictatorship.
Topic: Natural Born
Tecson v COMELEC
(This case digest is limited to the issue of natural-born as
provided in the case list.)

CO N F L I C T O F L AW S
Nationality & Domiciliary Theory |7
(This case is about W/N FPJ is a natural-born Filipino citizen.)

The conclusions that could be drawn with some degree of


certainty from the documents would be that -

Facts: On 31 December 2003, respondent Ronald Allan Kelly Poe,


also known as Fernando Poe, Jr. (hereinafter "FPJ"), in the
forthcoming national elections. In his certificate of candidacy,
FPJ, representing himself to be a natural-born citizen of the
Philippines, stated his name to be "Fernando Jr.," or "Ronald
Allan" Poe, his date of birth to be 20 August 1939 and his place
of birth to be Manila.
Victorino X. Fornier, initiated a petition before the Commission
on Elections ("COMELEC") to disqualify FPJ and to deny due
course or to cancel his certificate of candidacy upon the thesis
that FPJ made a material misrepresentation in his certificate of
candidacy by claiming to be a natural-born Filipino citizen when
in truth, according to Fornier, his parents were foreigners; his
mother, Bessie Kelley Poe, was an American, and his father,
Allan Poe, was a Spanish national, being the son of Lorenzo Pou,
a Spanish subject.
Granting, petitioner asseverated, that Allan F. Poe was a Filipino
citizen, he could not have transmitted his Filipino citizenship to
FPJ, the latter being an illegitimate child of an alien mother.
Petitioner based the allegation of the illegitimate birth of
respondent on two assertions - first, Allan F. Poe contracted a
prior marriage to a certain Paulita Gomez before his marriage to
Bessie Kelley and, second, even if no such prior marriage had
existed, Allan F. Poe, married Bessie Kelly only a year after the
birth of respondent.
Issue: W/N FPJ is a natural born citizen. YES
Held:
In ascertaining, in G.R. No. 161824, whether grave
abuse of discretion has been committed by the COMELEC, it is
necessary to take on the matter of whether or not respondent
FPJ is a natural-born citizen, which, in turn, depended on
whether or not the father of respondent, Allan F. Poe,
would have himself been a Filipino citizen and, in the
affirmative, whether or not the alleged illegitimacy of
respondent prevents him from taking after the Filipino
citizenship of his putative father.
Any conclusion on the Filipino citizenship of Lorenzo Pou could
only be drawn from the presumption that having died in 1954 at
84 years old, Lorenzo would have been born sometime in the
year 1870, when the Philippines was under Spanish rule, and
that San Carlos, Pangasinan, his place of residence upon his
death in 1954, in the absence of any other evidence, could have
well been his place of residence before death, such that
Lorenzo Pou would have benefited from the "en masse
Filipinization" that the Philippine Bill had effected in
1902.

1. The parents of FPJ were Allan F. Poe and Bessie


Kelley;
2. FPJ was born to them on 20 August 1939;
3. Allan F. Poe and Bessie Kelley were married to each
other on 16 September, 1940;
4. The father of Allan F. Poe was Lorenzo Poe; and
5. At the time of his death on 11 September 1954,
Lorenzo Poe was 84 years old.
The death certificate of Lorenzo Pou would indicate that he died
on 11 September 1954, at the age of 84 years, in San Carlos,
Pangasinan. It could thus be assumed that Lorenzo Pou was born
sometime in the year 1870 when the Philippines was still a
colony of Spain.
Petitioner would argue that Lorenzo Pou was not in the
Philippines during the crucial period of from 1898 to 1902
considering that there was no existing record about such fact in
the Records Management and Archives Office. Petitioner,
however, likewise failed to show that Lorenzo Pou was at any
other place during the same period. In his death certificate, the
residence of Lorenzo Pou was stated to be San Carlos,
Pangasinan.
In the absence of any evidence to the contrary, it should be
sound to conclude, or at least to presume, that the place of
residence of a person at the time of his death was also his
residence before death.
Topic: How Filipino Citizenship Acquired
IN RE petition to declare ZITA NGO to possess all
qualifications and none of the disqualifications for
naturalization under Commonwealth Act 473 for the
purpose of cancelling her alien registry with the BUREAU
OF IMMIGRATION.
ZITA NGO BURCA, petitioner and appellee,
G.R. No. L-24252
January 30, 1967
FACTS:

Zita Ngo was born in Gigaquit, Surigao (now Surigao del


Norte), on March 30, 1933. Her father was Ngo Tay Suy and
her mother was Dee See alias Lee Co, now both deceased
and citizens of Nationalist Republic of China. She was born
on March 30, 1933 in Gigaquit, Surigao and holds Native
Born Certificate of Residence 46333 and Alien Certificate of
Registration A-148054. She married Florencio Burca a
native-born Filipino, on May 14, 1961.

A petition was personally filed to declare Zita Ngo also


known as Zita Ngo Burca "as possessing all qualifications
and none of the qualifications for naturalization under
Commonwealth Act 473 for the purpose of cancelling her
Alien Registry with the Bureau of Immigration".

That citizenship (of Lorenzo Pou), if acquired, would thereby


extend to his son, Allan F. Poe, father of respondent FPJ.
The 1935 Constitution, during which regime respondent FPJ has
seen first light, confers citizenship to all persons whose fathers
are Filipino citizens regardless of whether such children
are legitimate or illegitimate.
(In case sir would ask how FPJs grandfather was considered a
Filipino citizen.)

CO N F L I C T O F L AW S
Nationality & Domiciliary Theory |8

Notice of hearing was sent to the Solicitor General and duly


published.

Solicitor General opposed and moved to dismiss the


petition on two main grounds, viz: (1) that "there is no
proceeding established by law, or the rules for the judicial
declaration of the citizenship of an individual"; and (2) that
as an application for Philippine citizenship, "the petition is
fatally defective for failure to contain or mention the
essential allegations required under Section 7 of the
Naturalization Law", such as, among others, petitioner's
former places of residence (that petitioner was born in
Gigaquit, Surigao that her former residence was Surigao,
Surigao, and that presently she is residing at Regal St.,
Ormoc City. In court, however, she testified that
she also resided in Junquera St., Cebu, where she took up a
course in home economics, for one year. Section 7 of the
Naturalization Law requires that a petition for naturalization
should state petitioner's "present and former places of
residence". ), and the absence of the affidavits of at least
two supporting witnesses.

Trial was held. Sole witness was petitioner. With the


documentary evidence admitted, the case was submitted
for decision.

Decision was rendered dismissing the opposition, and


declaring that ZITA NGO BURCA petitioner, has all the
qualifications and none of the disqualifications to become a
Filipino Citizen and that she being married to a Filipino
Citizen, was declared a citizen of the Philippines, after
taking the necessary oath of allegiance, as soon as this
decision becomes final and executory.

ISSUE: WON Zita Ngo, an alien woman, who married a Filipino


citizen by mere fact of marriage, automatically became
a Filipino citizen. NO. SHE HAS TO APPLY FOR
NATURALIZATION.
RULING:

By constitutional and legal precepts, an alien woman


who marries a Filipino citizen, does not by the
mere fact of marriage - automatically become a
Filipino citizen.

Thus, by Article IV of the Constitution, citizenship is limited


to:

Those who are citizens of the Philippine Islands at the time


of the adoption of this Constitution.

Those born in the Philippine Islands of foreign parents who,


before the adoption of this Constitution, had been elected
to public office in the Philippine Islands.

Those whose fathers are citizens of the Philippines.

Those whose mothers are citizens of the Philippines and,


upon reaching the age of majority, elect Philippine
citizenship.

Those who are naturalized in accordance with law.


And, on the specific legal status of an alien woman married
to a citizen of the Philippines, Congress in paragraph 1,

Section 15 of the Revised Naturalization Law legislated the


following:
o

Any woman who is now or may hereafter be married to a


citizen of the Philippines, and who might herself be lawfully
naturalized shall be deemed a citizen of the Philippines.

Jurisprudence uniformly pronouncement that an alien wife


of a Filipino citizen may not acquire the status of a citizen of
the Philippines unless there is proof that she herself may be
lawfully naturalized. Which means that, in line with the
national policy of selective admission to Philippine
citizenship, the wife must possess the qualifications under
Section 2, and must not be laboring under any of the
disqualifications enumerated in Section 4, of the Revised
Naturalization Law.

If an alien woman married to a Filipino does not


become ipso facto a citizen, then she must have to file
a "petition for citizenship" in order that she may acquire
the status of a Filipino citizen. Authority for this view is
Section 7 of the Revised Naturalization Law in which the
plain language is: "Any person desiring to acquire Philippine
citizenship, shall file with the competent court" a petition
for the purpose. And this, because such alien woman is not
a citizen, and she desires to acquire it. The proper forum,
Section 8 of the same law points out, is the Court of First
Instance of the province where the petitioner has resided
"at least one year immediately preceding the filing of the
petition".

We accordingly rule that: (1) An alien woman married to


a Filipino who desires to be a citizen of this country
must apply therefor by filing a petition for
citizenship reciting that she possesses all the
qualifications set forth in Section 2, and none of the
disqualifications under Section 4, both of the Revised
Naturalization Law; (2) Said petition must be filed in
the Court of First Instance where petitioner has
resided at least one year immediately preceding the
filing of the petition; and (3) Any action by any other
office, agency, board or official, administrative or
otherwise other than the judgment of a competent
court of justice certifying or declaring that an alien
wife of the Filipino citizen is also a Filipino citizen, is
hereby declared null and void.

Section
2. Qualifications.
Subject to section four of this Act,
any person having the following
qualifications may become a
citizen of the Philippines by
naturalization:
First. He must be not less than
twenty-one years of age on the
day of the hearing of the
petition;
Second. He must have resided in
the Philippines for a continuous
period of not less than ten
years;
Third. He must be of good moral
character and believes in the
principles
underlying
the
Philippine Constitution, and must
have conducted himself in a
proper
and
irreproachable
manner during the entire period

Section 4. Who are


disqualified. The
following cannot be
naturalized
as
Philippine citizens:
a.
Persons
opposed
to
organized
government
or
affiliated with any
association
or
group of persons
who uphold and
teach
doctrines
opposing
all
organized
governments;
b.
Persons
defending
or
teaching
the
necessity
or

CO N F L I C T O F L AW S
Nationality & Domiciliary Theory |9
of
his
residence
in
the
Philippines in his relation with
the constituted government as
well as with the community in
which he is living.
Fourth. He must own real estate
in the Philippines worth not less
than
five
thousand
pesos,
Philippine currency, or must
have some known lucrative
trade, profession, or lawful
occupation;
Fifth. He must be able to speak
and write English or Spanish and
any one of the principal
Philippine languages; and
Sixth. He must have enrolled his
minor children of school age, in
any of the public schools or
private schools recognized by
the Office of Private Education of
the
Philippines,
where
the
Philippine history, government
and
civics
are
taught
or
prescribed as part of the school
curriculum, during the entire
period of the residence in the
Philippines required of him prior
to the hearing of his petition for
naturalization
as
Philippine
citizen.

c.

d.

e.

f.

g.

h.

propriety
of
violence, personal
assault,
or
assassination
for
the success and
predominance
of
their ideas;
Polygamists or
believers in the
practice
of
polygamy;
Persons
convicted of crimes
involving
moral
turpitude;
Persons
suffering
from
mental alienation
or
incurable
contagious
diseases;
Persons who,
during the period
of their residence
in the Philippines,
have not mingled
socially with the
Filipinos, or who
have not evinced a
sincere desire to
learn and embrace
the
customs,
traditions,
and
ideals
of
the
Filipinos;
Citizens
or
subjects of nations
with
whom
the
United States and
the Philippines are
at war, during the
period of such war;
Citizens
or
subjects
of
a
foreign
country
other
than
the
United
States whose laws
do
not
grant
Filipinos the right
to
become
naturalized citizens
or subjects thereof.

MOY YA LIM YAO alias EDILBERTO AGUINALDO LIM and


LAU
YUEN
YEUNG, petitioners-appellants, vs.
THE
COMMISSIONER OF IMMIGRATION, respondent-appellee.
G.R. No. L-21289 October 4, 1971

FACTS:

On February 8, 1961, Lau Yuen Yeung applied for a passport


visa to enter the Philippines as a non-immigrant. In the
interrogation made in connection with her application for a
temporary visitor's visa to enter the Philippines, she stated
that she was a Chinese residing at Kowloon, Hongkong, and
that she desired to take a pleasure trip to the Philippines to
visit her great (grand) uncle Lau Ching Ping for a period of
one month. She was permitted to come into the Philippines
on March 13, 1961, and was permitted to stay for a period
of one month which would expire on April 13, 1961. On the
date of her arrival, Asher Y, Cheng filed a bond in the
amount of P1,000.00 to undertake, among others that said
Lau Yuen Yeung would actually depart from the Philippines
on or before the expiration of her authorized period of stay
in this country or within the period as in his discretion the
Commissioner
of
Immigration
or
his
authorized
representative might properly allow. After repeated
extensions, petitioner Lau Yuen Yeung was allowed to stay
in the Philippines up to February 13, 1962.

On January 25, 1962, she contracted marriage with Moy Ya


Lim Yao alias Edilberto Aguinaldo Lim an alleged Filipino
citizen. Because of the contemplated
action
of
Commissioner of Immigration to confiscate her bond and
order her arrest and immediate deportation, after the
expiration of her authorized stay, she brought this action for
injunction with preliminary injunction.

ISSUE:

WON Yeung ipso facto became a Filipino citizen upon her


marriage to a Filipino citizen. YES. (Please refer to the
fulltext for the discussion regarding the evolution of cases
from the first Ly Giok Ha case until this case was decided.)

RULING:

CONTENTION OF THE SOLGEN: Solicitor General implicitly


concedes that had it been established in the proceedings
below that appellant Lau Yuen Yeung possesses all the
qualifications required by the law of applicants for
naturalization, she would have been recognized by the
respondent as a Filipino citizen in the instant case, without
requiring her to submit to the usual proceedings for
naturalization.

The phrases "shall be deemed" "shall be considered," and


"shall automatically become" as used in the above
provision, are undoubtedly synonymous. The leading idea or
purpose of the provision was to confer Philippine citizenship
by operation of law upon certain classes of aliens as a legal
consequence of their relationship, by blood or by affinity, to
persons who are already citizens of the Philippines.
Whenever
the fact
of
relationship of
the
persons
enumerated in the provision concurs with the fact of
citizenship of the person to whom they are related, the
effect is for said persons to become ipso facto citizens of
the Philippines. "Ipso facto" as here used does not mean
that all alien wives and all minor children of Philippine
citizens, from the mere fact of relationship, necessarily
become such citizens also. Those who do not meet the
statutory requirements do not ipso facto become citizens;
they must apply for naturalization in order to acquire such
status. What it does mean, however, is that in respect
of those persons enumerated in Section 15, the relationship

CO N F L I C T O F L AW S
N a t i o n a l i t y & D o m i c i l i a r y T h e o r y | 10
to a citizen of the Philippines is the operative fact which
establishes the acquisition of Philippine citizenship by them.
Necessarily, it also determines the point of time at which
such citizenship commences. Thus, under the second
paragraph of Section 15, a minor child of a Filipino
naturalized under the law, who was born in the Philippines,
becomes ipso facto a citizen of the Philippines from the time
the fact of relationship concurs with the fact of citizenship
of his parent, and the time when the child became a citizen
does not depend upon the time that he is able to prove that
he was born in the Philippines. The child may prove some
25 years after the naturalization of his father that he was
born in the Philippines and should, therefore, be
"considered" a citizen thereof. It does not mean that he
became a Philippine citizen only at that later time. Similarly,
an alien woman who married a Philippine citizen may be
able to prove only some 25 years after her marriage
(perhaps, because it was only 25 years after the marriage
that her citizenship status became in question), that she is
one who might herself be lawfully naturalized." It is not
reasonable to conclude that she acquired Philippine
citizenship only after she had proven that she "might
herself be lawfully naturalized." It is not reasonable to
conclude that she acquired Philippine citizenship only after
she had proven that she "might herself be lawfully
naturalized."

The point that bears emphasis in this regard is that in


adopting the very phraseology of the law, the legislature
could not have intended that an alien wife should not be
deemed a Philippine citizen unless and until she proves that
she might herself be lawfully naturalized. Far from it, the
law states in plain terms that she shall be deemed a citizen
of the Philippines if she is one "who might herself be
lawfully naturalized." The proviso that she must be one
"who might herself be lawfully naturalized" is not a
condition precedent to the vesting or acquisition of
citizenship; it is only a condition or a state of fact necessary
to establish her citizenship as a factum probandum, i.e., as
a fact established and proved in evidence. The word
"might," as used in that phrase, precisely replies that at the
time of her marriage to a Philippine citizen, the alien woman
"had (the) power" to become such a citizen herself under
the laws then in force. (Owen v. Kelly, 6 DC 191 [1867], aff'd
Kelly v. Owen, 76 US 496, 19 L ed 283 [1869). That she
establishes such power long after her marriage does not
alter the fact that at her marriage, she became a citizen.
This Court has held that "an alien wife of a Filipino citizen
may not acquire the status of a citizen of the
Philippines unless there is proof that she herself may be
lawfully naturalized". Under this view, the "acquisition" of
citizenship by the alien wife depends on her having
proven her qualifications for citizenship, that is, she is not a
citizen unless and until she proves that she may herself be
lawfully naturalized. It is clear from the words of the
law that the proviso does not mean that she
must first prove that she "might herself be lawfully
naturalized" before she shall be deemed (by
Congress, not by the courts) a citizen. Even the
"uniform" decisions cited by this Court to support its holding
did not rule that the alien wife becomes a citizen
only after she has proven her qualifications for citizenship.
What those decisions ruled was that the alien wives in those
cases failed to prove their qualifications and therefore they
failed to establish their claim to citizenship. Thus in Ly Giok
Ha v. Galang, 101 Phil. 459 [l957], the case was remanded
to the lower court for determination of whether petitioner,
whose claim to citizenship by marriage to a Filipino was
disputed by the Government, "might herself be lawfully
naturalized," for the purpose of " proving her alleged
change of political status from alien to citizen" (at 464).

In Cua v. Board, 101 Phil. 521 [1957], the alien wife who
was being deported, claimed she was a Philippine citizen by
marriage to a Filipino. This Court finding that there was no
proof that she was not disqualified under Section 4 of the
Revised Naturalization Law, ruled that: "No such evidence
appearing on record, the claim of assumption of Philippine
citizenship by Tijoe Wu Suan, upon her marriage to
petitioner, is untenable." (at 523) It will be observed that in
these decisions cited by this Court, the lack of proof that
the alien wives "might (themselves) be lawfully naturalized"
did not necessarily imply that they did not become, in truth
and in fact, citizens upon their marriage to Filipinos. What
the decisions merely held was that these wives failed to
establish their claim to that status as a proven fact.

In all instances where citizenship is conferred by operation


of law, the time when citizenship is conferred should not be
confused with the time when citizenship status is
established as a proven fact. Thus, even a natural-born
citizen of the Philippines, whose citizenship status is put in
issue in any proceeding would be required to prove, for
instance, that his father is a citizen of the Philippines in
order to factually establish his claim to citizenship. His
citizenship status commences from the time of birth,
although his claim thereto is established as a fact only at a
subsequent time. Likewise, an alien woman who might
herself be lawfully naturalized becomes a Philippine
citizen at the time of her marriage to a Filipino
husband, not at the time she is able to establish that
status as a proven fact by showing that she might
herself be lawfully naturalized. Indeed, there is no
difference between a statutory declaration that a
person
is
deemed
a
citizen
of
the
Philippines provided his father is such citizen from a
declaration that an alien woman married to a Filipino
citizen of the Philippines provided she might herself
be lawfully naturalized. Both become citizens by
operation of law; the former becomes a citizen ipso
facto upon birth; the later ipso facto upon marriage.

It is true that unless and until the alien wife proves that she
might herself be lawfully naturalized, it cannot be said that
she has established her status as a proven fact. But neither
can it be said that on that account, she did not become a
citizen of the Philippines. If her citizenship status is not
questioned in any legal proceeding, she obviously has no
obligation to establish her status as a fact. In such a case,
the presumption of law should be that she is what she
claims to be. (U.S. v. Roxas, 5 Phil. 375 [1905]; Hilado v.
Assad, 51 O.G. 4527 [1955]). There is a presumption that a
representation shown to have been made is true. (Aetna
Indemnity Co. v. George A. Fuller, Co., 73 A. 738, 74 A. 369,
111 ME. 321).

IN VIEW OF ALL THE FOREGOING, the judgment of the Court a


quo dismissing appellants' petition for injunction is hereby
reversed and the Commissioner of Immigration and/or his
authorized representative is permanently enjoined from causing
the arrest and deportation and the confiscation of the bond of
appellant Lau Yuen Yeung, who is hereby declared to have
become a Filipino citizen from and by virtue of her marriage to
her co-appellant Moy Ya Lim Yao alias Edilberto Aguinaldo Lim, a
Filipino citizen on January 25, 1962. No cost
FELISA LEE alias LEE SIU LUAN assisted by her husband,
JACKSON
BARRA petitioners-appellees, vs.
COMMISSIONER OF IMMIGRATION, respondent-appellant.

CO N F L I C T O F L AW S
N a t i o n a l i t y & D o m i c i l i a r y T h e o r y | 11
G.R. No. L-23446 December 20, 1971

pursuant to section 4 of Commonwealth Act No. 473, as


amended.

FACTS:
2.

Felisa Lee, a Chinese citizen, married Jackson Barra, a


Filipino citizen. Claiming to have acquired the citizenship of
her husband by virtue of her marriage on the ground that
she possessed all the qualifications and none of the
disqualifications for naturalization as a Filipino citizen, she
applied to the Commissioner of Immigration for cancellation
of her Alien Certificate of Registration.

Commissioner informed her that her petition could not be


given due course by his Office "in view of its existing policy
of requiring wives of Filipino citizens to go to the courts for
judicial determination of whether or not they possess all the
qualifications and none of the disqualifications required by
law to acquire the Filipino citizenship of their husbands."

Lee filed in the RTC a petition for the cancellation of her


ACR.

Solicitor General moved to dismiss the petition on the


ground that the court had no jurisdiction over the case,
alleging that "under Section 44 (h) of the Judiciary Act of
1948, a Court of First Instance cannot issue a writ
of mandamus to compel performance of an act outside the
judicial district."

Once the Commissioner of Immigration cancels the subject's


registration as an alien, there will probably be less difficulty in
establishing her Filipino citizenship in any other proceeding,
depending naturally on the substance and vigor of the
opposition.
G.R. No. 99358January 30, 1995
DJUMANTAN, petitioner, vs. HON. ANDREA D. DOMINGO,
COMMISSIONER OF THE BOARD OF IMMIGRATION, HON.
REGINO R. SANTIAGO and HON. JORGE V. SARMIENTO,
COMMISSIONERS
BUREAU
OF
IMMIGRATION
AND
DEPORTATION, respondents.
FACTS:

ISSUE:
Should the petition be granted? YES.

RULING:

In a long line of decisions, this Court has repeatedly held


that there is no proceeding established by law, or the rules,
for the judicial declaration of the citizenship of an individual.
The question that keeps bouncing back as a consequence of
the foregoing views is, what substitute is there for
naturalization proceedings to enable the alien wife of a
Philippine citizen to have the matter of her own citizenship
settled and established so that she may not have to be
called upon to prove it everytime she has to perform an act
or enter into a transaction or business or exercise a right
reserved only to Filipinos?
The ready answer to such question is that as the
laws of our country, both substantive and
procedural, stand today, there is no such procedure,
but such paucity is no proof that the citizenship
under discussion is not vested as of the date of
marriage or the husband's acquisition of citizenship,
as the case may be, for the truth is that the same
situation obtains even as to native-born Filipinos.

Regarding the steps that should be taken by an alien


woman married to a Filipino citizen in order to acquire
Philippine citizenship, the procedure followed in the Bureau
of Immigration is as follows:

1.

The alien woman must file a petition for the cancellation of


her alien certificate of registration alleging, among other
things, that she is married to a Filipino citizen and that she
is not disqualified from acquiring her husband's citizenship

Upon the filing of said petition, which should be


accompanied and supported by the joint affidavit of the
petitioner and her Filipino husband to the effect that the
petitioner does not belong to any of the groups disqualified
by the cited section from becoming naturalized Filipino
citizen (please see attached CEB Form 1), the Bureau of
Immigration conducts an investigation and thereafter
promulgates its order or decision granting or denying the
petition.

Bernard Banez, the husband of Marina Cabael, went to


Indonesia as a contract worker.
In 1974, he embraced and was converted to Islam
thensubsequently married petitioner in accordance with
Islamic rites.
Banez returned to the Philippines in January 1979.
On January 13, 1979, petitioner and her two children with
Banez, (2-year old Marina and 9-month old Nikulas) arrived
in Manila as the "guests" of Banez. Banez made it appear
that he was just a friend of the family of petitioner and was
merely repaying the hospitability extended to him during
his stay in Indonesia.
When petitioner and her two children arrived at NAIA,
Banez, together with Marina Cabael, met them.
Banez executed an "Affidavit of Guaranty and Support," for
his "guests," stating among others that he was
guaranteeing petitioner and her two children who were
coming to Manila as temporary visitors out of gratitude for
their familys hospitality. [see FT for content of the
affidavit.]
Petitioner and her children were admitted to the Philippines
as temporary visitors under Section 9(a) of the Immigration
Act of 1940.
In 1981, Marina Cabael discovered the true relationship of
her husband and petitioner. She filed a complaint for
"concubinage" with the MTC against the two. Case was
dismissed.
On March 25, 1982, the immigration status of petitioner was
changed from temporary visitor to that of permanent
resident under Section 13(a) of the same law.
On April 14, 1982, petitioner was issued an alien certificate
of registration.
Not accepting the set-back, Banez' eldest son, Leonardo,
filed a letter complaint with the Ombudsman, who
subsequently referred the letter to the CID. On the basis of
the said letter, petitioner was detained at the CID detention
cell. She was later released pending the deportation
proceedings, after posting a cash bond.
Thereafter, she manifested to the CID that she be allowed
to depart voluntarily from the Philippines. However, she had
a change of heart and moved for the dismissal of the
deportation case on the ground that she was validly married
to a Filipino citizen.

CO N F L I C T O F L AW S
N a t i o n a l i t y & D o m i c i l i a r y T h e o r y | 12

On September 1990, the CID revoked her permanent


resident visa upon ruling that her marriage to Banez was
irregular and not in accordance with the PH laws.
Petitioners argument: She was validly married to Banez,
and that under CC, FC, the Muslim Code, husband and wife
are obliged to live together.

ISSUE:
1. WON petitioner was lawfully admitted into the country and
thereafter lawfully acquired permanent residency. NO
2. WON petitioner should be deported. NO
HELD: {Note: Actually ang related langsa topic is yung one
sentence langnasinabi ng SC na Marriage of an alien woman to
a Filipino husband does not ipso facto make her a Filipino citizen
and does not excuse her from her failure to depart from the
country upon the expiration of her extended stay here as an
alien.}
1st Issue: SC said that it need not resolve the validity of
petitioner's marriage to Banez, if under the law the CID can
validly deport petitioner as an "undesirable alien" regardless of
her marriage to a Filipino citizen. Therefore, to be first resolved
is the question on petitioner's immigration status, particularly
the legality of her admission into the country and the change of
her status from temporary visitor to permanent resident. Upon a
finding that she was not lawfully admitted into the country and
she did not lawfully acquire permanent residency, the next
question is whether the power to deport her has prescribed.
There was a blatant abuse of our immigration laws in effecting
petitioner's entry into the country and the change of her
immigration status from temporary visitor to permanent
resident.
All such
privileges
were obtained
through
misinterpretation.
Never was the marriage of petitioner to Banez disclosed to the
immigration authorities in her applications for temporary
visitor's visa and for permanent residency.The civil status of an
alien applicant for admission as a temporary visitor is a matter
that could influence the exercise of discretion on the part of the
immigration authorities. The immigration authorities would be
less inclined to allow the entry of a woman who claims to have
entered into a marriage with a Filipino citizen, who is married to
another woman.
Generally, the right of the President to expel or deport aliens
whose presence is deemed inimical to the public interest is as
absolute and unqualified as the right to prohibit and prevent
their entry into the country. This right is based on the fact that
since the aliens are not part of the nation, their admission into
the territory is a matter of pure permission and simple tolerance
which creates no obligation on the part of the government to
permit them to stay.The interest, which an alien has in being
admitted into or allowed to continue to reside in the country, is
protected only so far as Congress may choose to protect it.

visitor's visa. Once admitted into the country, the alien has no
right to an indefinite stay. Under Section 13 of the law, an alien
allowed to stay temporarily may apply for a change of status
and "may be admitted" as a permanent resident. Among those
considered qualified to apply for permanent residency is the wife
or husband of a Philippine citizen. The entry of aliens into
the country and their admission as immigrants is not a
matter of right, even if they are legally married to
Filipino citizens.
2nd Issue:The right of public respondents to deport petitioner
has prescribed, based on Section 37(b) of the Immigration Act of
1940. Provision says that deportation may be effected under
clauses 2, 7, 8, 11 and 12 of paragraph (a) of Sec. 37 at any
time after entry, but shall not be effected under any clause
unless the arrest in the deportation proceedings is made within
five years after the cause for deportation arises.Petitioner
was admitted and allowed entry into the Philippines on January
13, 1979 on the basis of false and misleading statements in her
application and in the other supporting documents submitted to
the immigration authorities. Leonardo Banez first complained
with the CID on November 19, 1980 about the manner
petitioner was admitted into the country and asked for her
deportation. Tolling the prescriptive period from November 19,
1980, when Leonardo C. Banez informed the CID of the illegal
entry of petitioner into the country, more than five years had
elapsed before the issuance of the order of her
deportation on September 27, 1990.
SC here reversed CIDs decision revoking the issuance of the
permanent resident visa to petitioner.
G.R. No. L-27429 August 27, 1969
IN THE MATTER OF THE PETITION FOR ADMISSION AS
CITIZEN OF THE PHILIPPINES.
OH HEK HOW, petitioner appellee, vs. REPUBLIC OF THE
PHILIPPINES, oppositor-appellant.
FACTS:

The fact of marriage by an alien to a citizen does not withdraw


her from the operation of the immigration laws governing the
admission and exclusion of aliens.Marriage of an alien
woman to a Filipino husband does not ipso facto make
her a Filipino citizen and does not excuse her from her
failure to depart from the country upon the expiration of
her extended stay here as an alien.
Under Section 9 of the Immigration Act of 1940, it is not
mandatory for the CID to admit any alien who applies for a

OH HEK HOW petitioned for naturalization as citizen of the


Philippines.
A decision granting his petition was rendered on January 16,
1964.
On January 17, 1966, Oh Hek How filed a motion alleging
that he had complied with the requirements of R.A. 530 and
praying that he be allowed to take his oath of allegiance
as such citizen and issued the corresponding certificate of
naturalization.
Upon Oh Hek How's testimony taken on February 9, 1966
the date set for the hearing of said motion, CFI issued an
order authorizing the taking of said oath. On that same
date, petitioner took it and the certificate of
naturalization was issued to him.
The Government seasonably gave notice of its intention to
appeal from said order and filed its record on appeal. Before
the same was approved, it also moved to cancel Oh Hek
How's certificate of naturalization, on the ground that it
was issued and the oath taken before the February 9,
1966 order had become final and executory.
CFI granted the motion on October 3, 1966, but, at the
same time, authorized the taking of a new oath by Oh Hek
How and the issuance in his favor of another certificate of
naturalization, after 30 days from notice to the SolGen.
Thereafter, or on November 26, 1966, the court approved
the record on appeal and, once more, authorized Oh Hek
How to "take a new or proper oath to validate the first one
made on February 9, 1966."
The case is now before the SC on said record on appeal filed
by the Government.

CO N F L I C T O F L AW S
N a t i o n a l i t y & D o m i c i l i a r y T h e o r y | 13
ISSUE:
1. WON the Oath of Allegiance taken by Oh Hek How was valid.
NO
2. WON he was qualified to become a naturalized PH citizen. NO
HELD: Oath of Allegiance was null and void
The oath of allegiance taken by petitioner on November 28,
1966, and the certificate of naturalization issued to him in
pursuance thereof, as well as the authority given therefor by the
lower court, are null and void. Indeed, the order of February 9,
had not and up to the present has not become final and
executory in view of the appeal duly taken by the Government.

countries shall not produce loss or forfeiture of his Philippine


citizenship, if the law of that country grants the same privilege
to its citizens and such had been agreed upon by treaty
between the Philippines and the foreign country from which
citizenship is acquired.
The question of how a Chinese citizen may strip himself
of that status is necessarily governed pursuant to
Articles 15 and 16 of our Civil Code by the laws of
China, not by those of the Philippines.As a consequence, a
Chinese national cannot be naturalized as a citizen of the
Philippines, unless he has complied with the laws of
Nationalist China requiring previous permission of its
Minister of the Interior for the renunciation of
nationality.

What is more, petitioner's second oath was taken, not only after
the filing of the notice of appeal and the submission of the
record on appeal, but also after the approval thereof. In other
words, the lower court had already lost its jurisdiction
over the case.
SC discussed Oh Hek Hows qualifications and declared
his certificate of naturalization and oath of allegiance
null and void:
Oh Hek Hows net income was not lucrative
Petitioner's net income in 1960 and 1961 was P3,945.65 and
P5,105.79, respectively. His ITR for 1962, filed subsequently to
the institution of this case, showed a net income of P6,485.50
for that year.
Considering that petitioner has a wife and 3 children, one of
them of school age, at the time of the filing of his application for
naturalization, his aforementioned income is not a lucrative one.
Indeed, it has been held that the following incomes are not
lucrative, from the viewpoint of our naturalization laws, namely:
(1) P4,200 or P5,000 a year for one married, with five (5)
children; (2) P6,000 a year for one married, with two (2) minor
children; and (3) P6,000 6 or P6,300 a year for one married, with
only one (1) child. (Based on jurisprudence)
Oh Hek How did not get the required permission from
China

G.R. No. L-61565 August 20, 1990


REPUBLIC OF THE PHILIPPINES, petitioner, vs. HON.
SOFRONIO SAYO, Presiding Judge of the CFI of Nueva Vizcaya,
Branch I, and RAMON TAN BIANA JR., respondents.
FACTS:

Petitioner has not obtained from the Minister of the Interior of


Nationalist China the permission required by the laws thereof for
a valid renunciation of his Chinese citizenship.
It is argued that the same is not required by our laws and that
the naturalization of an alien, as a citizen of the Philippines, is
governed exclusively by such laws and cannot be controlled by
any foreign law.
Section 12 of Commonwealth Act No. 473 provides, however,
that before the naturalization certificate is issued, the petitioner
shall "solemnly swear," inter alia, that he renounces "absolutely
and forever all allegiance and fidelity to any foreign prince,
potentate" and particularly to the state "of which" he is "a
subject or citizen." The obvious purpose of this requirement is to
divest him of his former nationality, before acquiring Philippine
citizenship, because, otherwise, he would have two nationalities
and owe allegiance to two (2) distinct sovereignties, which our
laws do not permit, except that, pursuant to R.A. 2639, "the
acquisition of citizenship by a natural-born Filipino citizen from
one of the Iberian and any friendly democratic Ibero-American

Ramon Tan Biana, Jr. was born on January 9, 1952 in Solano,


Nueva Vizcaya, as the 5th legitimate child of the spouses
Ramon Tan Biana and Tiu Muy. His birth was registered on
the same day in the Office of the Local Civil Registrar of
Solano, Nueva Vizcaya, by the nurse who attended to his
birth. He claims that, in the process, the attending nurse
erroneously reported to the Local Civil Registrar that Ramon
Jr.'s citizenship, and the citizenship of his parents, as
"Chinese" instead of "Filipino". According to him, his "true
and real citizenship", and that of his parents, is Philippine
citizenship.
On February 1982, Ramon Jr. filed a petition before the then
CFIBayombong, Nueva Vizcaya, for the correction of entries
in the Civil Registry of Solano, Nueva Vizcaya, relating to his
citizenship and the citizenship of his legitimate parents, as
appearing in his Certificate of Birth.
Copies of the petition were furnished to the Office of the
Provincial Fiscal, the Office of the SolGen and the LCR.
Notice of hearing to be set on April 15, 1982 was issued, a
copy which was served on the aforesaid offices. A copy of
the notice of hearing was also posted by the Sheriff in the
bulletin board of the townhall of Solano, Nueva Vizcaya. The
notice of hearing was also published in the "Vizcaya
Advocate", a newspaper of general circulation published in
Solano, Nueva Vizcaya, once a week for three (3)
consecutive weeks.
At the initial hearing of the petition on April 15, no
appearance was entered by the Office of the SolGen,
despite its receipt of a copy of the petition and of the notice
of hearing.In view of the failure of a representative of the
SolGen to appear, the trial court reset the hearing of the
petition to May 14, 1982 and in an Order of the court,
requested the SolGen to file either an opposition, comment
or any other responsive pleading to the petition since the
court believed that the petition was significant, involving, as
it does, a change of citizenship. Still, no pleading was
received by the trial court from the SolGens office.
Accordingly, the trial court proceeded to receive evidence
for Ramon Jr. on May 14, with the Office of the Provincial
Fiscal representing the Government.

CO N F L I C T O F L AW S
N a t i o n a l i t y & D o m i c i l i a r y T h e o r y | 14

After hearing and presentation of evidence, the trial court


ordered the LCR to make in the entries and records the
corrections sought by Ramon Jr. so as to reflect a change in
the citizenship of Ramon Tan Biana, Jr. as well as his parents
Ramon Tan Biana and Tiu Muy from "Chinese" to "Filipino"
and to furnish copies of the corrected Certificate of Birth to
"all other offices concerned."
On August 4, 1982, Office of SolGen finally appeared and
filed MR, arguing that the entries ordered corrected by the
court were "not merely clerical of a harmless or innocuous
nature but involved substantial matters which should not
have been decided in "a merely summary proceeding" but
rather in "an appropriate action wherein all parties who may
be affected by the entries are notified or represented."
Denied, hence this petition.
Ramon Jr. counters that he does not seek a judicial
declaration of his citizenship but rather merely a correction
of an entry in the LCRs Office as to his citizenship and that
of his parents, considering that the citizenship of his parents
had already been passed upon by the Bureau of
Immigration. He further contends that the proceedings
taken before the trial court were not summary in nature,
and that the decision was rendered only after the required
notices had been given and after a hearing.

ISSUE: WON an appropriate adversary proceeding was done.


YES
HELD: The Court considers that the procedure followed in the
case at bar satisfied the requirements of "appropriate adversary
pro-proceedings."
SC cited Republic vs. Valencia, where it held:
It is undoubtedly true that if the subject matter of a
petition is not for the correction of clerical errors of a harmless
and innocuous nature, but one involving nationality or
citizenship, which is indisputably substantial as well as
controverted, affirmative relief cannot be granted in a
proceeding summary in nature. However, it is also true that a
right in law may be enforced and a, wrong may be remedied
as long as the appropriate remedy is used. This Court
adheres to the principle that even substantial errors in a civil
registry may be corrected and the true facts established
provided the parties aggrieved by the error avail
themselves
of
the
appropriate
adversary
proceeding.xxx
What is meant by 'appropriate adversary
proceeding?' Blacks Law Dictionary defines 'adversary
proceeding' as follows:One having opposing parties;
contested, as distinguished from an exparte application, one
of which the party seeking relief has given legal warning to
the other party, and afforded the latter an opportunity to
contest it. Excludes an adoption proceeding.
Thus, the persons who must be made parties to a
proceeding concerning the cancellation or correction of an
entry in the civil register are
(1) the civil registrar, and
(2) all persons who have or claim any interest which
would be affected thereby.
Upon the filing of the petition, it becomes the duty of
the court to issue an order fixing the time and place for the
hearing of the petition, and (2) cause the order for hearing to
be published once a week for three (3) consecutive weeks in a

newspaper of general circulation in the province. The


following are likewise entitled to oppose the petition the civil
registrar, and (2) any person having or claiming any interest
under the entry whose cancellation or correction is sought.
If all these procedural requirements have been
followed, a petition for correction and /or cancellation of
entries in the record of birth even if filed and conducted under
Rule 108 of the Revised Rules of Court can no longer be
described as 'summary'. There can be no doubt that when an
opposition to the petition is filed either by the Civil Registrar
or any person having or claiming any interest in the entries
sought to be cancelled and/or corrected and the opposition is
actively prosecuted, the proceedings thereon become
adversary proceedings.
Tested by the standards projected in Valencia, the
proceedings taken in the instant case appear to be
appropriate adversary proceedings.
As indicated earlier, a copy of the petition was sent
both to the Office of the Nueva Vizcaya Provincial Fiscal and the
OSG. The petition was set for hearing and a notice of hearing
was served upon the Office of the Provincial Fiscal LCR and OSG,
posted by the Sheriff in the bulletin board of the municipal
townhall, and published once a week for three (3) consecutive
weeks in a newspaper of general circulation in Solano, Nueva
Vizcaya.Upon failure of the OSG to appear at the first hearing
set by the trial court, the latter reset the hearing for the
succeeding month and expressly requested the SolGen to
appear and to file a responsive pleading considering that the
changes in the LCR requested were "significant" involving the
citizenship of Ramon Jr. Notwithstanding the resetting of the
hearing, and despite having received notice of the rescheduled
hearing, OSG entered no appearance and filed no pleading
before the trial court. The Assistant Provincial Fiscal appeared on
behalf of the Government, was present when the court
proceeded to receive evidence for the petitioner on the
rescheduled hearing,and did not file any opposition to the
petition. It must be assumed that the Assistant Provincial Fiscal
did not do so because he saw no need to file such an opposition.
SC does not believe that his failure to file an opposition changed
the adversarial character of the proceedings. It certainly was not
Ramon Jr.'s fault that neither the Assistant Provincial Fiscal nor
anyone else saw fit to oppose the petition for correction.
Both in the MR filed by the SolGen of the decision of the trial
court and in the Petition for Review filed before this Court,
SolGen did not question the genuineness, authenticity,
relevancy or sufficiency of the evidence submitted before the
trial court relating to Ramon Jr.'s citizenship and that of his
parents. SolGen limited itself to contending that substantial
changes of entries in the LCR are not to be made pursuant to "a
summary proceeding."
REPUBLIC OF THE PHILIPPINES, petitioner, vs.LEONOR
VALENCIA, as Natural mother and guardian of her minor
children, BERNARDO GO and JESSICA GO; and THE HON.
AGAPITO HONTANOSAS, Judge of the COURT OF FIRST INSTANCE
OF CEBU, Branch XI. G.R. No. L-32181 March 5, 1986
FACTS: Respondent Leonor Valencia, for and in behalf of her
minor children, Bernardo Go and Jessica Go filed with the CFI of
Cebu a petition for the cancellation and/or correction of entries
of birth of her minor children in the Civil Registry of Cebu City.
The SolGen opposed the petition alleging that the petition for
correction of entry in the Civil Registry pursuant to Article 412 of
the New Civil Code of the Philippines in relation to Rule 108 of

CO N F L I C T O F L AW S
N a t i o n a l i t y & D o m i c i l i a r y T h e o r y | 15
the Revised Rules of Court, contemplates a summary proceeding
and correction of mere clerical errors, those harmless and
innocuous changes such as the correction of a name that is
merely mispelled, occupation of parents, etc., and not changes
or corrections involving civil status, nationality, or citizenship
which are substantial and controversial.
Finding the petition to be sufficient in form and substance, the
trial court then issued an order directing the publication of the
petition and the date of hearing thereof.
Valencia, filed her reply to the opposition wherein she alleged
that substantial changes in the civil registry records involving
the civil status of parents, their nationality or citizenship may be
allowed if- (1) the proper suit is filed, and (2) evidence is
submitted, either to support the allegations of the petition or to
disprove the same; that respondents have complied with these
requirements by filing the present special proceeding for
cancellation or correction of entries in the civil registry pursuant
to Rule 108 of the Revised Rules of Court and that they have
caused reasonable notice to be given to the persons named in
the petition and have also caused the order for the hearings of
their petition to be published for three (3) consecutive weeks in
a newspaper of general circulation in the province.
Subsequently, the Local Civil Registrar of Cebu City filed a
motion to dismiss on the ground that since the petition seeks to
change the nationality or citizenship of Bernardo Go and Jessica
Go from "Chinese" to "Filipino" and their status from
"Legitimate" to Illegitimate", and changing also the status of the
mother from "married" to "single" the corrections sought are not
merely clerical but substantial, involving as they do the
citizenship and status of the petitioning minors and the status of
their mother.
RTC: denied the motion to dismiss. After trial on the merits, it
rendered a decision ordering the Local Civil Registrar of the City
of Cebu to make the necessary cancellation and/or correction on
the following entries in the Record of Birth of the children.
The petitioner premises its case on precedents from the 1954
case of Ty Kong Tin v. Republic (94 Phil. 321) to the 1981 case
of Republic v. Caparosso (107 SCRA 67), that entries which can
be corrected under Article 412 of the New Civil Code as
implemented by Rule 108 of the Revised Rules of Court refer to
those mistakes that are clerical in nature or changes that are
harmless and innocuous (Wong v. Republic, 115 SCRA 496).
ISSUE: WON the correction of citizenship and civil status of the
petitioner and that of her minor children was proper.
HELD: YES.

Article 412 thereof which prescribes judicial order before an


entry in a civil register shall be changed or corrected. This
requirement was deemed necessary to forestall the commission
of fraud or other mischief in these matters.
But even then, it is not any correction that can be considered
under Article 412 of he Civil Code. The nature of the corrections
sought has to be considered and if found to refer only to clerical
errors the same may be allowed under said article which was
construed to contemplate only a summary proceeding.
And so in the Ty Kong Tin case, this Honorable Court took
occasion to draw a distinction between what entries in the civil
register could be corrected under Article 412 of the New Civil
Code and what could not. In the process, to our mind, this
Honorable Court set down propositions which hold true not only
in that case but also in the subsequent cases for the latter
merely reiterated the Ty Kong Tin decision. These are:
First, that proceedings under Article 412 of the New Civil Code
are summary:
Second, that corrections in the entires in the civil register may
refer to either mere mistakes that are clerical in nature or
substantial ones which affects the civil status or -the nationality
or citizenship of the persons involved; and
Third, that if the change or correction sought refers to mere
correction of mistakes that are clerical in nature the same may
be done, under Article 412 of the Civil Code; otherwise, if it
refers to a substantial change which affects the civil status or
citizenship of a party. the matter should be threshed out in a
proper action.
To our humble estimation, these propositions do not
altogether bar or preclude substantial changes or
corrections involving such details as the civil status or
nationality of a party. As a matter of fact, just three years
after the Ty Kong Tin decision, this Honorable Court allowed a
party to correct mistakes involving such substantial matters as
his birthplace and citizenship in the birth certificates of his two
sons. (Lim v. Republic, No. L-8932, May 31, 1957, 101 Phil. 1235)
Only that where the correction pertains to matters which are
important and controversial certain conditions sine que non
have to be complied with. Thus it was held:
If it refers to a substantial change which affects the status or
citizenship of a party, the matter should be threshed out in a
proper action ... .' (Ty Kong Tin v. Republic, supra)

(**I included the discussion with respect to the procedural issue


just in case i-ask ni Sir)

It will thus be gleaned from the foregoing that corrections


involving such matters as the civil status of the parents, their
nationality or citizenship may be allowed provided the proper
suit is filed.

Changes or corrections in the entries in the civil registry were


governed, at first, by Act No. 3753 (Civil Registry Law) which
placed these matters exclusively upon the sound judgment and
discretion of the civil registrars. With the effectivity of the New
Civil Code on August 30, 1950, these matters were governed by

In the instant case, a petition for cancellation and/or correction


of entries of birth of Bernardo Go and Jessica Go in the Civil
Registry of the City of Cebu was filed by respondent Leonor
Valencia on January 27, 1970, and pursuant to the order of the
trial court dated February 4, 1970, the said petition was

CO N F L I C T O F L AW S
N a t i o n a l i t y & D o m i c i l i a r y T h e o r y | 16
published once a week for three (3) consecutive weeks in
the, Cebu Advocate, a newspaper of general circulation in the
City of Cebu. Notice thereof was duly served on the Solicitor
General. the Local Civil Registrar and Go Eng. The order likewise
set the case for hearing and directed the local civil registrar and
the other respondents or any person claiming any interest under
the entries whose corrections were sought, to file their
opposition to the said petition. An opposition to the petition was
consequently filed by the Republic on February 26, 1970.
Thereafter a full blown trial followed with respondent Leonor
Valencia testifying and presenting her documentary evidence in
support of her petition. The Republic on the other hand crossexamined respondent Leonor Valencia.
We are of the opinion that the petition filed by the respondent in
the lower court by way of a special proceeding for cancellation
and/or correction of entries in the civil register with the requisite
notice and publication and the recorded proceedings that
actually took place thereafter could very well be regarded as
that proper suit or appropriate action.
To follow the petitioner's argument that Rule 108 is not
an appropriate proceeding without in any way
intimatingwhat is the correct proceeding or if such a
proceeding exists at all, would result in manifest
injustice.
Apart from Bernardo Go and Jessica Go, there are four
(4) other sisters and one (1) other brother born of the
same father and mother. Not only are all five registered
as Filipino citizens but they have pursued careers which
require Philippine citizenship as a mandatory prerequisite. To emphasize the strict policy of the government
regarding professional examinations, it was the law until
recently that to take the board exams for pharmacist, the
applicant should possess natural born citizenship. (See. 18,
Republic Act 5921 and Sec. 1, P.D. 1350)
The sisters and brother are:
1. Sally Go, born on April 29, 1934 was licensed as a Pharmacist
after passing the government board examinations in 1956.
2. Fanny Go, born on July 12, 1936 is a Registered Nurse who
passed the government board examinations in 1960.
3. Corazon Go, born on June 20, 1939, during the trial of this
case in 1970 was a fourth year medical student, qualified to
take the government board examinations after successfully
completing the requirements for a career in medicine, and
presumably is a licensed physician now.
4. Antonio Go, born February 14, 1942 was an engineering
student during the 1970 trial of the case and qualified by
citizenship to take government board examinations.
5. Remedios Go, born October 4, 1945 was a licensed
Optometrist after passing the government board examinations
in 1967.

The above facts were developed and proved during trial. The
petitioner failed to refute the citizenship of the minors
Bernardo and Jessica Go.
In this petition, it limits itself to a procedural reason to overcome
substantive findings by arguing that the proper procedure was
not followed.
There are other facts on the record. Leonor Valencia is a
registered voter and had always exercised her right of suffrage
from the time she reached voting age until the national
elections immediately preceding the filing of her petition. The
five other sisters and brother are also registered voters and
likewise exercised the right of suffrage.
An uncle of the mother's side had held positions in the
government having been elected twice as councilor and twice as
vice-mayor of Victorias, Negros Occidental. Respondent Leonor
Valencia has purchased and registered two (2) parcels of land as
per Transfer Certificate of Title No. T-46104 and Transfer
Certificate of Title No. T-37275. These allegations are well
documented and were never contradicted by the Republic. As
correctly observed by the lower court.
The right of suffrage is one of the important rights of a
citizen. This is also true with respect to the acquisition of
a real property. The evidence further shows that her
children had been allowed to take the Board
Examinations given by the Government for Filipino
citizens only.
It would be a denial of substantive justice if two children
proved by the facts to be Philippine citizens, and whose
five sisters and brother born of the same mother and
father enjoy all the rights of citizens, are denied the
same rights on the simple argument that the "correct
procedure" not specified or even intimated has not been
followed.
We are, therefore, constrained to deny the petition.
G.R. No. 195649
July 2, 2013
CASAN
MACODE
MACQUILING,
vs.COMMISSION ON ELECTIONS, ROMMEL
CAGOCO, AND LINOG G. BALUA. RESPONDENTS.

PETITIONER,
ARNADO Y

Facts:
Rommel Arnado is a natural born Filipino citizen.
However, as a consequence of his subsequent naturalization as
an American citizen he lost his Filipino citizenship. Arnado
applied for repatriation under R.A. 9225 before the Consulate
General of the Philippines in San Franciso, USA and took the
Oath of Allegiance to the Republic of the Philippines on 10 July
2008. On the same day an Order of Approval of his Citizenship
Retention and Re-acquisition was issued in his favor.
On 3 April 2009 Arnado again took his Oath of Allegiance to the
Republic and executed an Affidavit of Renunciation of his foreign
citizenship. On 30 November 2009, Arnado filed his Certificate of
Candidacy for Mayor of Kauswagan, Lanao del Norte.
Linog C. Balua (Balua), another mayoralty candidate, filed a
petition to disqualify Arnado and/or to cancel his certificate of
candidacy. Balua contended that Arnado is not a resident of
Kauswagan, Lanao del Norte and that he is a foreigner,

CO N F L I C T O F L AW S
N a t i o n a l i t y & D o m i c i l i a r y T h e o r y | 17
attaching thereto a certification issued by the Bureau of
Immigration dated 23 April 2010 indicating the nationality of
Arnado as "USA-American." To further bolster his claim of
Arnados US citizenship, Balua presented in his Memorandum a
computer-generated travel recordindicating that Arnado has
been using his US Passport in entering and departing the
Philippines. The said record shows that Arnado left the country
on 14 April 2009 and returned on 25 June 2009, and again
departed on 29 July 2009, arriving back in the Philippines on 24
November 2009.
Issue: WON Arnado reacquired his citizenship?
Ruling: Yes. The use of foreign passport after renouncing ones
foreign citizenship is a positive and voluntary act of
representation as to ones nationality and citizenship; it does not
divest Filipino citizenship regained by repatriation but it recants
the Oath of Renunciation required to qualify one to run for an
elective position.
Indeed, Arnado took the Oath of Allegiance not just only once
but twice: first, on 10 July 2008 when he applied for repatriation
before the Consulate General of the Philippines in San Francisco,
USA, and again on 03 April 2009 simultaneous with the
execution of his Affidavit of Renunciation. By taking the Oath of
Allegiance to the Republic, Arnado re-acquired his Philippine
citizenship. At the time, however, he likewise possessed
American citizenship. Arnado had therefore become a dual
citizen.
After reacquiring his Philippine citizenship, Arnado renounced his
American citizenship by executing an Affidavit of Renunciation,
thus completing the requirements for eligibility to run for public
office.
By renouncing his foreign citizenship, he was deemed to be
solely a Filipino citizen, regardless of the effect of such
renunciation under the laws of the foreign country.
However, this legal presumption does not operate permanently
and is open to attack when, after renouncing the foreign
citizenship, the citizen performs positive acts showing his
continued possession of a foreign citizenship.
Arnado himself subjected the issue of his citizenship to attack
when, after renouncing his foreign citizenship, he continued to
use his US passport to travel in and out of the country before
filing his certificate of candidacy on 30 November 2009. The
pivotal question to determine is whether he was solely and
exclusively a Filipino citizen at the time he filed his certificate of
candidacy, thereby rendering him eligible to run for public office.
Between 03 April 2009, the date he renounced his foreign
citizenship, and 30 November 2009, the date he filed his COC,
he used his US passport four times, actions that run counter to
the affidavit of renunciation he had earlier executed. By using
his foreign passport, Arnado positively and voluntarily
represented himself as an American, in effect declaring before
immigration authorities of both countries that he is an American
citizen, with all attendant rights and privileges granted by the
United States of America.
The renunciation of foreign citizenship is not a hollow oath that
can simply be professed at any time, only to be violated the
next day. It requires an absolute and perpetual renunciation of
the foreign citizenship and a full divestment of all civil and
political rights granted by the foreign country which granted the
citizenship.

We agree with the COMELEC En Banc that such act of using a


foreign passport does not divest Arnado of his Filipino
citizenship, which he acquired by repatriation. However, by
representing himself as an American citizen, Arnado voluntarily
and effectively reverted to his earlier status as a dual citizen.
Such reversion was not retroactive; it took place the instant
Arnado represented himself as an American citizen by using his
US passport.
This act of using a foreign passport after renouncing ones
foreign citizenship is fatal to Arnados bid for public office, as it
effectively imposed on him a disqualification to run for an
elective local position.
REPUBLIC OF THE PHILIPPINES vs. AZUCENA SAAVEDRA
BATUGAS
October 7, 2013
FACTS:
On December 2, 2002, Azucena filed a Petition
for Naturalization before the RTC of Zamboanga del Sur.
Azucena was born in Zamboanga del Sur on September 28,
1941 to Chinese parents. Her primary, secondary, and tertiary
education were taken in Philippine schools. She then practiced
her teaching profession in various schools situated in the
Philippines. In 1968, at the age of 26, Azucena married Santiago
Batuigas (Santiago),a natural-born Filipino citizen. They have
five children.
SolGen filed its Motion to Dismiss on the ground that
Azucena failed to allege that she is engaged in a lawful
occupation or in some known lucrative trade. The motion was
denied and the hearing for the reception of Azucenas evidence
was then set.
Neither the OSG nor the Office of the Provincial
Prosecutor appeared on the day of the hearing. Azucenas
counsel moved that the evidence be presented ex-parte, which
the RTC granted. Accordingly, the RTC designated its Clerk of
Court as Commissioner to receive Azucenas evidence.During
the November 5, 2004 ex-parte hearing, no representative from
the OSG appeared despite due notice.
RTC RULING : Azucena has amply supported the
allegations in her Petition and none of the disqualifications to be
admitted as citizen of the Philippines in accordance with the
provisions of the Naturalization Law.
OSG argued that the ex-parte presentation of evidence
before the Branch Clerk of Court violates Section 10 of CA 473,
as the law mandates public hearing in naturalization cases.
RTC rejected the argument. It held that the public has
been fully apprised of the naturalization proceedings and was
free to intervene. The OSG and its delegate, the Provincial
Prosecutor, are the only officers authorized by law to appear on
behalf of the State, which represents the public. Thus, when the
OSG was furnished with a copy of the notice of hearing for the
reception of evidence ex-parte, there was already a sufficient
compliance with the requirement of a public hearing.
OSG then appealed the RTC judgment to the CA,
contending that Azucena failed to comply with the income
requirement under CA 473. The OSG maintained that Azucena is
not allowed under the Retail Trade Law (Republic Act No. 1180)
to engage directly or indirectly in the retail trade. Hence, she
cannot possibly meet the income requirement. And even if she
is allowed, her business is not a "lucrative trade" within the
contemplation of the law or that which has an appreciable

CO N F L I C T O F L AW S
N a t i o n a l i t y & D o m i c i l i a r y T h e o r y | 18
margin of income over expenses in order to provide for
adequate support in the event of unemployment, sickness, or
disability to work. The OSG likewise disputed Azucenas claim
that she owns real property because aliens are precluded from
owning lands in the country.
The OSG further asserted that the ex-parte proceeding
before the commissioner is not a "public hearing" as ex-parte
hearings are usually done in chambers, without the public in
attendance. It claimed that the State was denied its day in court
because the RTC, during the May 18, 2004 initial hearing,
immediately allowed the proceeding to be conducted ex-parte
without even giving the State ample opportunity to be present.
Azucena countered that although she is a teacher by
profession, she had to quit to help in the retail business of her
husband, and they were able to send all their children to
school.It is highly unlikely that she will become a public charge
as she and her spouse have enough savings and could even be
given sufficient support by their children. She contended that
the definition of "lucrative trade/income" should not be strictly
applied to her. Being the wife and following Filipino tradition, she
should not be treated like male applicants for naturalization who
are required to have their own "lucrative trade."
Azucena denied that the hearing for her Petition was
not made public, as the hearing before the Clerk of Court was
conducted in the courts session hall. Besides, the OSG cannot
claim that it was denied its day in court as notices have always
been sent to it. Hence, its failure to attend is not the fault of the
RTC.
CA RULING: OSGs appeal dismissed. CA found that Azucenas
financial condition permits her and her family to live with
reasonable comfort in accordance with the prevailing standard
of living and consistent with the demands of human dignity.
CA held that the RTC had complied with the mandate of
the law requiring notice to the OSG and the Provincial Prosecutor
of its scheduled hearing for the Petition.
ISSUE: WON the petition for naturalization should be
granted? YES.
HELD: Foreign women who are married to Philippine
citizens may be deemed ipso facto Philippine citizens
Under existing laws, an alien may acquire Philippine citizenship
through either judicial naturalization under CA 473 or
administrative naturalization under Republic Act No. 9139 (the
"Administrative Naturalization Law of 2000"). A third option,
called derivative naturalization, which is available to alien
women married to Filipino husbands is found under Section 15 of
CA 473, which provides that:
"any woman who is now or may hereafter be married to a citizen
of the Philippines and who might herself be lawfully naturalized
shall be deemed a citizen of the Philippines."
Under this provision, foreign women who are married to
Philippine citizens may be deemed ipso facto Philippine citizens
and it is neither necessary for them to prove that they possess
other qualifications for naturalization at the time of their
marriage nor do they have to submit themselves to judicial
naturalization. Copying from similar laws in the United States
which has since been amended, the Philippine legislature
retained Section 15 of CA 473, which then reflects its intent to
confer Filipino citizenship to the alien wife thru derivative
naturalization.

PROPER PROCEDURE: As stated in Moy Ya Lim Yao, the


procedure for an alien wife to formalize the conferment of
Filipino citizenship is as follows:
Regarding the steps that should be taken by an alien woman
married to a Filipino citizen in order to acquire Philippine
citizenship, the procedure followed in the Bureau of
Immigration is as follows: The alien woman must file a
petition for the cancellation of her alien certificate of registration
alleging, among other things, that she is married to a Filipino
citizen and that she is not disqualified from acquiring her
husbands citizenship pursuant to Section 4 of Commonwealth
Act No. 473, as amended. Upon the filing of said petition, which
should be accompanied or supported by the joint affidavit of the
petitioner and her Filipino husband to the effect that the
petitioner does not belong to any of the groups disqualified by
the cited section from becoming naturalized Filipino citizen x x
x, the Bureau of Immigration conducts an investigation and
thereafter promulgates its order or decision granting or denying
the petition.( NOTE: in this case, Azuzena followed such
procedure but it was denied [by ministry of justice] so she filed
the petition for naturalization).
In this case: However, the case before us is a Petition for
judicial naturalization and is not based on Section 15 of CA 473
which was denied by the then Ministry of Justice. The lower
court which heard the petition and received evidence of her
qualifications and absence of disqualifications to acquire
Philippine citizenship, has granted the Petition, which was
affirmed by the CA.
Azucena is a teacher by profession and has actually exercised
her profession before she had to quit her teaching job to assume
her family duties and take on her role as joint provider, together
with her husband, in order to support her family. Together,
husband and wife were able to raise all their five children,
provided them with education, and have all become
professionals and responsible citizens of this country. Certainly,
this is proof enough of both husband and wifes lucrative
trade. Azucena herself is a professional and can resume
teaching at anytime. Her profession never leaves her, and this is
more than sufficient guarantee that she will not be a charge to
the only country she has known since birth.
Judicial declaration of citizenship (not authorized) vs
naturalization proceeding
This case however is not a Petition for judicial declaration of
Philippine citizenship but rather a Petition for judicial
naturalization under CA 473. In the first, the petitioner believes
he is a Filipino citizen and asks a court to declare or confirm his
status as a Philippine citizen. In the second, the petitioner
acknowledges he is an alien, and seeks judicial approval to
acquire the privilege of be coming a Philippine citizen based on
requirements required under CA 473.Azucena has clearly
proven, under strict judicial scrutiny, that she is qualified for the
grant of that privilege, and this Court will not stand in the way of
making her a part of a truly Filipino family.
Topic: How Filipino Citizenship Lost
BOARD OF IMMIGRATION COMMISSIONERS and
COMMISSIONER OF IMMIGRATION, petitioners, vs. BEATO
GO CALLANO, MANUEL GO CALLANO, GONZALO GO
CALLANO, JULIO GO CALLANO and THE COURT OF
APPEALS, respondents. October 31, 1968
FACTS: Petitioners, Go Callano brothers, are illegitimate
children of Emilia Callano, a Filipino citizen, with her commonlaw husband a Chinese citizen. All were born in Leyte,

CO N F L I C T O F L AW S
N a t i o n a l i t y & D o m i c i l i a r y T h e o r y | 19
Philippines. In 1946, they, together with their parents, went to
Amoy, China. In 1961, they applied with the Philippine Consul
General in Hongkong for entry into the Philippines as Filipino
citizens. The Consulate made thereafter the appropriate
investigation, and on the basis of evidence presented issued a
certificate of registration and identity to the effect that the
applicant had submitted sufficient evidence of their citizenship
and identity and had been allowed to register in the Consulate
as Filipino citizens and to travel directly to the Philippines. On
December 26 of the same year 1961, they arrived in Manila.
On August 21, 1962, the Board of Immigration
Commissioners, without any previous notice and hearing,
reversed the decision admitting Beato and his three brothers for
entry as citizens of the Philippines and ordered their exclusion
as aliens not properly documented for admission, as their cable
authorization No. 2230-V (File No. 23617) authorizing the
documentation of Beato Go Callano and others, were not
authentic. The BOI Commissioner also ordered that they be
returned to the port where they came or to the country of which
they were nationals. The Commissioner of Immigration then
issued a warrant of exclusion pursuant to the said order.
The GO Callano brothers subsequently filed an action
for injunction to restrain the Board of Immigration
Commissioners and the Commissioner of Immigration from
executing the exclusion order. CFI issued a writ of preliminary
injunction restraining the respondents from deporting the
petitioners. NOTWITHSTANDING, the Court held that the
petitioners are citizens of the Republic of China. The
grounds upon which the Court based its decision were:
(1) because petitioners stayed in China for a period of fifteen
years before returning to the Philippines, they must be
considered as citizens of the Chinese Republic;
(2) as petitioners were recognized by their alien father as his
children, they became Chinese citizens under the Chinese law of
nationality.
While the Court also found that the cable authorization
mentioned heretofore was a forgery, it held that, for the purpose
of the petition before it, "it was immaterial to determine the
genuineness or falsity of the cable authorization. For if the
petitioners are Filipino citizens, they are entitled to remain
within the territorial jurisdiction of the Republic in whatever way
they might have entered." CA reversed.
Hence, the Board of Immigration Commissioners and the
Commissioner of Immigration filed this appeal maintain, among
others, petitioners are aliens and that, granting that they were
Filipino citizens when they left the Philippines in 1946, they lost
that citizenship, firstly, by staying in China for a period of fifteen
years, and secondly, because they were recognized by their
common-law father, they became citizens of the Republic of
China in accordance with the Chinese Nationality Law.
ISSUE: WON petitioners lost their Filipino citizenship by staying
in China for a period of fifteen years, and because they were
recognized by their common-law father.
HELD: NO
The following portions of the decision of the Court of Appeals
would seem to be sufficient answer:
The question, whether petitioners who are admittedly Filipino
citizens at birth subsequently acquired Chinese citizenship
under the Chinese Law of Nationality by reason of

recognition or a prolonged stay in China, is a fit subject for


the Chinese law and the Chinese court to determine, which
cannot be resolved by a Philippine court without encroaching
on the legal system of China. For, the settled rule of
international law, affirmed by the Hague Convention on
Conflict of Nationality Laws of April 12, 1930 and by the
International Court of Justice, is that "Any question as to
whether a person possesses the nationality of a particular
state should be determined in accordance with laws of that
state ." (quoted in Salonga, Private International Law, 1957
Ed., p. 112.) There was no necessity of deciding that
question because so far as concerns the petitioners' status,
the only question in this proceeding is: Did the petitioners
lose their Philippine citizenship upon the performance of
certain acts or the happening of certain events in China? In
deciding this question no foreign law can be applied. The
petitioners are admittedly Filipino citizens at birth, and their
status must be governed by Philippine law wherever they
may be, in conformity with Article 15 (formerly Article 9) of
the Civil Code which provides as follows: "Laws relating to
family rights and duties, or to the status, conditions and
legal capacity of persons are binding upon citizens of the
Philippines, even though living abroad." Under Article IV,
Section 2, of the Philippine Constitution, "Philippine
citizenship may be lost or reacquired in the manner provided
by law," which implies that the question of whether a Filipino
has lost his Philippine citizenship shall be determined by no
other than the Philippine law.
Section 1 of Commonwealth Act No. 63, as amended by
Republic Act No. 106, provides that a Filipino citizen may lose his
citizenship by naturalization in a foreign country; express
renunciation of citizenship; subscribing to an oath of allegiance
to support the constitution or laws of a foreign country;
rendering service to, or accepting a commission in, the armed
forces of a foreign country; cancellation of the certificate of
naturalization; declaration by competent authority that he is a
deserter of the Philippine armed forces in time of war; in the
case of a woman by marriage to a foreigner if, by virtue of laws
in force in her husband's country, she acquires his nationality.
Recognition of the petitioners by their alien father is not among
the ground for losing Philippine citizenship under Philippine law,
and it cannot be said that the petitioners lost their former status
by reason of such recognition. About the only mode of losing
Philippine citizenship which closely bears on the petitioners is
renunciation. But even renunciation cannot be cited in support
of the conclusion that petition lost their Philippine citizenship
because the law requires an express renunciation which means
a renunciation that is made known distinctly and explicitly and
not left to inference or implication; a renunciation manifested by
direct and appropriate language, as distinguished from that
which is inferred from conduct. (Opinion No. 69 of the Secretary
of Justice, Series of 1940.)
Indeed, as the Supreme Court held in U.S. v. Ong Tianse, 29 Phil.
332, a case for deportation, where Ong, a natural child of a
Filipino mother and a Chinese father, born in the Philippines, was
brought by his parents to China when he was 4 years old, where
he remained for 18 or 19 years, returning to the Philippines at
25 years of age, "The fact that a minor child in those conditions
was taken to China and remained there for several years is not
sufficient ground upon which to hold that he has changed his
nationality, when, after reaching his majority, he did not express
his desire to choose the nationality of his father." The import of
the foregoing pronouncement is that of itself a protracted stay in
a foreign country does not amount to renunciation. Moreover,
herein petitioners were all minors when they where brought to
China in 1446. They were without legal capacity to renounce
their status. Upon their return to the Philippines only Beato Go
Callano had attained the age of majority, but even as to him
there could not have been renunciation because he did not
manifest by direct and appropriate language that he was
disclaiming Philippine citizenship. On the contrary, after he has

CO N F L I C T O F L AW S
N a t i o n a l i t y & D o m i c i l i a r y T h e o r y | 20
attained the age of majority, he applied for registration as a
Philippine citizen and sought entry into this country, which are
clear indicia of his intent to continue his former status. The
foregoing shows that the petitioners have not lost their
Philippine citizenship.
IN RE PETITION FOR HABEAS CORPUS OF WILLIE YU,
petitioner,
vs.
MIRIAM DEFENSOR-SANTIAGO, BIENVENIDO P. ALANO,
JR., MAJOR PABALAN, DELEO HERNANDEZ, BLODDY
HERNANDEZ, BENNY REYES and JUN ESPIRITU SANTO,
respondent.

reacquisition of Portuguese citizenship is grossly inconsistent


with his maintenance of Philippine citizenship.
Denial, if any, of due process was obviated when
petitioner was given by the Court the opportunity to show proof
of continued Philippine citizenship, but he has failed.
While normally the question of whether or not a person
has renounced his Philippine citizenship should be heard before
a trial court of law in adversary proceedings, this has become
unnecessary as this Court, no less, upon the insistence of
petitioner, had to look into the facts and satisfy itself on whether
or not petitioner's claim to continued Philippine citizenship is
meritorious.

Facts:

Petitioner was originally issued a Portuguese passport in


1971 valid for 5 yrs and renewed for the same period upon
presentment before the proper Portuguese consular officer.
Despite his naturalization as a Philippine citizen on Feb 10
1978, on July 21 1981, petitioner applied for and was issued
Portuguese Passport by the Consular Section of the
Portuguese Embassy in Tokyo.
While still a citizen of the Philippines who had renounced,
upon his naturalization, absolutely and forever all
allegiance and fidelity to any foreign prince, potentate,
state or sovereignty and pledged to maintain true faith
and allegiance to the Republic of the Philippines, he
declared his nationality as Portuguese in commercial
documents he signed, specifically, the Companies registry
of Tai Shun Estate, Ltd. filed in Hong Kong sometime in April
1980.

Philippine citizenship, it must be stressed, is not a


commodity or were to be displayed when required and
suppressed when convenient.

there are 3 dissenting opinions:

petitioner was divested of his Filipino citizenship through a


SUMMARY PROCEDURE. Fernan
I disagree with the SUMMARY PROCEDURE employed in this
case to divest a Filipino of his citizenship Guttierez
..do not constitute substantial evidence..Cortes
basahalangnnyo if you have time

Procedural facts:

Petition for habeas corpus filed for release from


detention of petitioner Yu
Petition denied
TRO issued against respondent Commissioner to cease
and desist from immediately deporting Yu pending the
conclusion of hearings before the Board of Special
Inquiry, CID (Commission on Immigration and
Deportation)
Respondent Commissioner filed motion to lift TRO on
the basis that a summary judgment of deportation
against Yu was issued by the CID Board of
Commissioners.
To finally dispose the case, the Supreme Court ruled on
the several motions and gave petitioner non-extendible
3 days within which to explain and prove why he should
still be considered a citizen of the Philippines despite
his acquisition and use of a Portuguese passport.

Issue: WON Filipino citizenship of petitioner Yu was lost by virtue


of his acquisition and use of Portuguese passport. YES
Held: The foregoing acts considered together constitute an
express renunciation of petitioners Philippine citizenship
acquired through naturalization.In Board of Immigration
Commissioners us, Go Gallano, express renunciation was held to
mean a renunciation that is made known distinctly and explicitly
and not left to inference or implication. Petitioner, with full
knowledge, and legal capacity, after having renounced
Portuguese citizenship upon naturalization as a Philippine citizen
resumed or reacquired his prior status as a Portuguese citizen,
applied for a renewal of his Portuguese passport and
represented himself as such in official documents even after he
had become a naturalized Philippine citizen. Such resumption or

COQUILLA vs. COMELEC


FACTS: - In 1938, Coquilla was born of Filipino parents in Oras,
Eastern Samar.
- In 1965, he joined the United States Navy and was
subsequently naturalized as a US citizen.
- He remained in the US but made several trips to the
Philippines.
- In November 10, 2000, he applied for repatriation under RA
8171 and his application was approved. He then took his oath as
a citizen of the Philippines. Subsequently, he applied for
registration as a voter of Samar.
- In 2001, he filed his certificate of candidacy stating therein
that he had been a resident of Oras for 2 years.
- Alvarez, who was the incumbent mayor and who was running
for reelection, sought the cancellation of Coquillas certificate of
candidacy on the ground that the latter made a material
representation in his COC by stating that he had been a resident
of Oras for 2 years (at least 1 year immediately preceding the
day of the election is required) when in truth he had resided
therein for only about 6 months since November 10, 2000 when
he took his oath as a citizen of the Philippines (Nov 2000-May
2001).
ISSUE: WON Coquillas COC should be cancelled. YES.
RULING: Coquilla lost his domicile/residence of Oras by
becoming a US citizen after enlisting in the US Navy. From then
on and until November 10, 2000, when he reacquired Phil.
citizenhship, Coquilla was a non-resident alien.
Coquilla cannot contend that he was compelled to
adopt American citizenship only by reason of his service in the
US armed forces. It is noteworthy that Coquilla was repatriated

CO N F L I C T O F L AW S
N a t i o n a l i t y & D o m i c i l i a r y T h e o r y | 21
not under RA 2630, which applies to the repatriation of those
who lost their Phil. Citizenship by accepting commission in the
AF of US but under RA 8171, which as earlier mentioned,
provides for repatriation of, among others, natural-born Filipinos
who lost their citizenship on account of political or economic
necessity. In any event, the fact is that, by having been
naturalized abroad, he lost his Phil. Citizenship and with
it his residence in the Philippines. Until his reacquisition of
Phil. citizenship on November 10, 2000, Coquilla did not
reacquire his legal residence in this country.
CIRILO R. VALLES, petitioner, vs. COMMISSION ON
ELECTIONS and ROSALIND YBASCO LOPEZ, respondents.
[G.R. No. 137000. August 9, 2000]
[sorry long digest. Daghan siyag topics na ma igo :( ]
Facts:Rosalind Ybasco Lopez was born on May 16, 1934 in
Napier Terrace, Broome, Western Australia, to the spouses,
Telesforo Ybasco, a Filipino citizen and native of Daet, Camarines
Norte, and Theresa Marquez, an Australian. In 1949, at the age
of fifteen, she left Australia and came to settle in the Philippines.
On June 27, 1952, she was married to Leopoldo Lopez, a Filipino
citizen, at the Malate Catholic Church in Manila. Since then, she
has continuously participated in the electoral process not only
as a voter but as a candidate, as well. She served as Provincial
Board Member of the Sangguniang Panlalawigan of Davao
Oriental. In 1992, she ran for and was elected governor of Davao
Oriental. Her election was contested by her opponent, Gil Taojo,
Jr., in a petition for quo warranto, docketed as EPC No. 92-54,
alleging
as
ground
therefor
her
alleged
Australian
citizenship.The same controversy arose in the 1995 local
elections, when respondent Rosalind Ybasco Lopez ran for reelection as governor of Davao Oriental. Her opponent, Francisco
Rabat, filed a petition for disqualification contesting her Filipino
citizenship but the said petition was likewise dismissed. The
citizenship of private respondent was once again raised as an
issue when she ran for re-election as governor of Davao Oriental
in the May 11, 1998 elections. Her candidacy was questioned by
the herein petitioner, Cirilo Valles,.
The Commission on Elections ruled that private respondent
Rosalind Ybasco Lopez is a Filipino citizen and therefore,
qualified to run for a public office because (1) her father,
Telesforo Ybasco, is a Filipino citizen, and by virtue of the
principle of jus sanguinis she was a Filipino citizen under the
1987 Philippine Constitution; (2) she was married to a Filipino,
thereby making her also a Filipino citizen ipso jure under Section
4 of Commonwealth Act 473; (3) and that, she renounced her
Australian citizenship on January 15, 1992 before the
Department of Immigration and Ethnic Affairs of Australia and
her Australian passport was accordingly cancelled as certified to
by the Australian Embassy in Manila; and (4) furthermore, there
are the COMELEC Resolutions in EPC No. 92-54 and SPA Case No.
95-066, declaring her a Filipino citizen duly qualified to run for
the elective position of Davao Oriental governor.
Petitioner, on the other hand, maintains that the private
respondent is an Australian citizen, placing reliance on the
admitted facts that:
a) In 1988, private respondent registered herself with the
Bureau of Immigration as an Australian national and
was issued Alien Certificate of Registration No. 404695
dated September 19, 1988;
b) On even date, she applied for the issuance of an
Immigrant Certificate of Residence (ICR), and
c) c) She was issued Australian Passport No. H700888 on
March 3, 1988.

Petitioner theorizes that under the aforestated facts and


circumstances, the private respondent had renounced her
Filipino citizenship. He contends that in her application for alien
certificate of registration and immigrant certificate of residence,
private respondent expressly declared under oath that she was
a citizen or subject of Australia; and said declaration forfeited
her Philippine citizenship, and operated to disqualify her to run
for elective office.
As regards the COMELECs finding that private respondent had
renounced her Australian citizenship on January 15, 1992 before
the Department of Immigration and Ethnic Affairs of Australia
and had her Australian passport cancelled on February 11, 1992,
as certified to by the Australian Embassy here in Manila,
petitioner argues that the said acts did not automatically restore
the status of private respondent as a Filipino citizen. According
to petitioner, for the private respondent to reacquire Philippine
citizenship she must comply with the mandatory requirements
for repatriation under Republic Act 8171; and the election of
private respondent to public office did not mean the restoration
of her Filipino citizenship since the private respondent was not
legally repatriated. Coupled with her alleged renunciation of
Australian citizenship, private respondent has effectively
become a stateless person and as such, is disqualified to run for
a public office in the Philippines; petitioner concluded.
Issue:Whether or not respondent is a Filipino citizen. YES
Held: (With regard to laws governing filipino citizenship)
The Philippine law on citizenship adheres to the principle of jus
sanguinis. Thereunder, a child follows the nationality or
citizenship of the parents regardless of the place of his/her birth,
as opposed to the doctrine of jus soli which determines
nationality or citizenship on the basis of place of birth.
Private respondent Rosalind Ybasco Lopez was born on May 16,
1934 in Napier Terrace, Broome, Western Australia, to the
spouses, Telesforo Ybasco, a Filipino citizen and native of Daet,
Camarines Norte, and Theresa Marquez, an Australian.
Historically, this was a year before the 1935 Constitution took
into effect and at that time, what served as the Constitution of
the Philippines were the principal organic acts by which the
United States governed the country. These were the Philippine
Bill of July 1, 1902 and the Philippine Autonomy Act of August
29, 1916, also known as the Jones Law.
Among others, these laws defined who were deemed to be
citizens of the Philippine islands. The Philippine Bill of 1902 and
the Jones Law, (two organic laws) provide that all inhabitants of
the Philippines who were Spanish subjects on April 11, 1899 and
resided therein including their children are deemed to be
Philippine citizens. Private respondents father, Telesforo Ybasco,
was born on January 5, 1879 in Daet, Camarines Norte, a fact
duly evidenced by a certified true copy of an entry in the
Registry of Births. Thus, under the Philippine Bill of 1902 and the
Jones Law, Telesforo Ybasco was deemed to be a Philippine
citizen. By virtue of the same laws, which were the laws in force
at the time of her birth, Telesforos daughter, herein private
respondent Rosalind Ybasco Lopez, is likewise a citizen of the
Philippines.
The signing into law of the 1935 Philippine Constitution has
established the principle of jus sanguinis as basis for the
acquisition of Philippine citizenship.
So also, the principle of jus sanguinis, which confers citizenship
by virtue of blood relationship, was subsequently retained under
the 1973 and 1987 Constitutions. Thus, the herein private
respondent, Rosalind Ybasco Lopez, is a Filipino citizen, having
been born to a Filipino father. The fact of her being born in
Australia is not tantamount to her losing her Philippine

CO N F L I C T O F L AW S
N a t i o n a l i t y & D o m i c i l i a r y T h e o r y | 22
citizenship. If Australia follows the principle of jussoli, then at
most, private respondent can also claim Australian citizenship
resulting to her possession of dual citizenship.
(As to respondents alleged renunciation of her Filipino
Citizenship)
Petitioner also contends that even on the assumption that the
private respondent is a Filipino citizen, she has nonetheless
renounced her Philippine citizenship. To buttress this contention,
petitioner cited private respondents application for an Alien
Certificate of Registration (ACR) and Immigrant Certificate of
Residence (ICR), on September 19, 1988, and the issuance to
her of an Australian passport on March 3, 1988.
Under Commonwealth Act No. 63, a Filipino citizen may lose his
citizenship:
(1)
By
naturalization
in
a
foreign
country;
(2)
By
express
renunciation
of
citizenship;
(3) By subscribing to an oath of allegiance to support the
constitution or laws of a foreign country upon attaining twentyone
years
of
age
or
more;
(4) By accepting commission in the military, naval or air service
of
a
foreign
country;
(5) By cancellation of the certificate of naturalization;
(6) By having been declared by competent authority, a deserter
of the Philippine armed forces in time of war, unless
subsequently, a plenary pardon or amnesty has been granted:
and
(7) In case of a woman, upon her marriage, to a foreigner if, by
virtue of the laws in force in her husbands country, she acquires
his nationality.
In order that citizenship may be lost by renunciation,
such renunciation must be express. Petitioners contention
that the application of private respondent for an alien certificate
of registration, and her Australian passport, is bereft of merit.
The mere fact that private respondent Rosalind Ybasco Lopez
was a holder of an Australian passport and had an alien
certificate of registration are not acts constituting an effective
renunciation of citizenship and do not militate against her claim
of Filipino citizenship. For renunciation to effectively result inthe
loss of citizenship, the same must be express. As held by this
court in the aforecited case of Aznar, an application for an alien
certificate of registration does not amount to an express
renunciation or repudiation of ones citizenship. The application
of the herein private respondent for an alien certificate of
registration, and her holding of an Australian passport, as in the
case of Mercado vs. Manzano, were mere acts of assertion of her
Australian citizenship before she effectively renounced the
same. Thus, at the most, private respondent had dual
citizenship - she was an Australian and a Filipino, as well.
Moreover, under Commonwealth Act 63, the fact that a child of
Filipino parent/s was born in another country has not been
included as a ground for losing ones Philippine citizenship. Since
private respondent did not lose or renounce her Philippine
citizenship, petitioners claim that respondent must go through
the process of repatriation does not hold water.
(As to sec 40 RA 7160 which disqualifies persons wth dual
citizenship from running for any elective office)
Petitioner also maintains that even on the assumption that the
private respondent had dual citizenship, still, she is disqualified
to run for governor of Davao Oriental; citing Section 40 of
Republic Act 7160 otherwise known as the Local Government
Code of 1991,
In the aforecited case of Mercado vs. Manzano, the Court
clarified dual citizenship as used in the Local Government Code
and reconciled the same with Article IV, Section 5 of the 1987

Constitution on dual allegiance. Recognizing situations in which


a Filipino citizen may, without performing any act, and as an
involuntary consequence of the conflicting laws of different
countries, be also a citizen of another state, the Court explained
that dual citizenship as a disqualification must refer to citizens
with dual allegiance.
Thus, the fact that the private respondent had dual citizenship
did not automatically disqualify her from running for a public
office. Furthermore, it was ruled that for candidates with
dual citizenship, it is enough that they elect Philippine
citizenship upon the filing of their certificate of
candidacy, to terminate their status as persons with dual
citizenship.This is so because in the certificate of candidacy,
one declares that he/she is a Filipino citizen and that he/she will
support and defend the Constitution of the Philippines and will
maintain true faith and allegiance thereto. Such declaration,
which is under oath, operates as an effective renunciation of
foreign citizenship. Therefore, when the herein private
respondent filed her certificate of candidacy in 1992, such fact
alone terminated her Australian citizenship.

Topic: How Filipino Citizenship Reacquired


REPUBLIC v DELA ROSA
FACTS:
This is a petition for certiorari under Rule 45 of the Revised
Rules of Court in relation to R.A. No. 5440 and Section 25 of the
Interim Rules, filed by the Republic of the Philippines: (1) to
annul the Decision dated February 27, 1992 of the Regional Trial
Court, Branch 28, Manila, in SP Proc. No. 91-58645, which readmitted private respondent as a Filipino citizen under the
Revised Naturalization Law (C.A. No. 63 as amended by C.A. No.
473); and (2) to nullify the oath of allegiance taken by private
respondent on February 27, 1992.
On September 20, 1991, petitioner filed a petition for
naturalization captioned: "In the Matter of Petition of Juan G.
Frivaldo to be Re-admitted as a Citizen of the Philippines
The respondent Judge set the petition for hearing on March 16,
1992, and directed the publication of the said order and petition
in the Official Gazette and a newspaper of general circulation,
for three consecutive weeks, the last publication of which should
be at least six months before the said date of hearing.
On January 14, 1992, private respondent filed a "Motion to Set
Hearing Ahead of Schedule," that it be done on January instead
of having it on March where he manifested his intention to run
for public office in the May 1992 elections. The motion was
granted and the hearing was moved on February.

CO N F L I C T O F L AW S
N a t i o n a l i t y & D o m i c i l i a r y T h e o r y | 23
Six days later, on February 27, respondent
assailed decision and held that petitioner
admitted as a ctitzen of the Republic of
naturalization, thereby vesting upon him,
priviliges of a natural born Filipino citizen.

judge rendered the


Juan Frivaldo is rethe Philippines by
all the rights and

After receiving a copy of the Decision on March 18, 1992, the


Solicitor General filed a timely appeal directly with the Supreme
Court.
ARGUMENTS OF FRIVALDO:
In his comment to the States appeal of the decision granting
him Philippine citizenship in G.R. No. 104654, Frivaldo alleges
that the precarious political atmosphere in the country during
Martial Law compelled him to seek political asylum in the United
States, and eventually to renounce his Philippine citizenship.
Naturalization as his chosen remedy
He claims that his petition for naturalization was his only
available remedy for his reacquisition of Philippine citizenship.
He tried to reacquire his Philippine citizenship through
repatriation and direct act of Congress. However, he was later
informed that repatriation proceedings were limited to army
deserters or Filipino women who had lost their citizenship by
reason of their marriage to foreigners. His request to Congress
for sponsorship of a bill allowing him to reacquire his Philippine
citizenship
failed
to
materialize,
notwithstanding
the
endorsement of several members of the House of
Representatives in his favor. He attributed this to the maneuvers
of his political rivals.

Naturalization as his chosen remedy


Frivaldo, having opted to reacquire Philippine citizenship thru
naturalization under the Revised Naturalization Law, is duty
bound to follow the procedure prescribed by the said law. It is
not for an applicant to decide for himself and to select the
requirements which he believes, even sincerely, are applicable
to his case and discard those which be believes are inconvenient
or merely of nuisance value. The law does not distinguish
between an applicant who was formerly a Filipino citizen and
one who was never such a citizen. It does not provide a special
procedure for the reacquisition of Philippine citizenship by
former Filipino citizens akin to the repatriation of a woman who
had lost her Philippine citizenship by reason of her marriage to
an alien.
Jurisdiction not acquired; Publication and posting of the petition
not complied
The trial court never acquired jurisdiction to hear the petition for
naturalization of private respondent. The proceedings
conducted, the decision rendered and the oath of allegiance
taken therein, are null and void for failure to comply with the
publication and posting requirements under the Revised
Naturalization Law.
Under Section 9 of the said law, both the petition for
naturalization and the order setting it for hearing must be
published once a week for three consecutive weeks in the
Official Gazette and a newspaper of general circulation.
Compliance therewith is jurisdictional (Po Yi Bo v. Republic, 205
SCRA 400 [1992]). Moreover, the publication and posting of the
petition and the order must be in its full test for the court to
acquire jurisdiction.

Rescheduling of Hearing to an earlier date


He also claims that the re-scheduling of the hearing of the
petition to an earlier date, without publication, was made
without objection from the Office of the Solicitor General. He
makes mention that on the date of the hearing, the court was
jam-packed.
It is Frivaldos posture that there was substantial compliance
with the law and that the public was well-informed of his petition
for naturalization due to the publicity given by the media.
Mandatory two-year waiting period
Anent the issue of the mandatory two-year waiting period prior
to the taking of the oath of allegiance, Frivaldo theorizes that
the rationale of the law imposing the waiting period is to grant
the public an opportunity to investigate the background of the
applicant and to oppose the grant of Philippine citizenship if
there is basis to do so. In his caseFrivaldo alleges that such
requirement may be dispensed with, claiming that his life, both
private and public, was well-known. Frivaldo cites his
achievement as a freedom fighter and a former Governor of the
Province of Sorsogon for six terms.

Absence of allegations required by the law to be contained in


the petition
The petition for naturalization lacks several allegations required
by Sections 2 and 6 of the Revised Naturalization Law,
particularly: (1) that the petitioner is of good moral character;
(2) that he resided continuously in the Philippines for at least ten
years; (3) that he is able to speak and write English and any one
of the principal dialects; (4) that he will reside continuously in
the Philippines from the date of the filing of the petition until his
admission to Philippine citizenship; and (5) that he has filed a
declaration of intention or if he is excused from said filing, the
justification therefor.
The absence of such allegations is fatal to the petition.
Likewise, the petition is not supported by the affidavit of at least
two credible persons who vouched for the good moral character
of private respondent as required by Section 7 of the Revised
Naturalization Law. Private respondent also failed to attach a
copy of his certificate of arrival to the petition as required by
Section 7 of the said law.
The proceedings was marred by several irregularities

ISSUE: Whether or not Frivaldo was duly re-admitted to his


citizenship
as
a
Filipino.
RULING: No.

The proceedings of the trial court was marred by the following


irregularities: (1) the hearing of the petition was set ahead of
the scheduled date of hearing, without a publication of the order

CO N F L I C T O F L AW S
N a t i o n a l i t y & D o m i c i l i a r y T h e o r y | 24
advancing the date of hearing, and the petition itself; (2) the
petition was heard within six months from the last publication of
the petition; (3) petitioner was allowed to take his oath of
allegiance before the finality of the judgment; and (4) petitioner
took his oath of allegiance without observing the two-year
waiting period.

Facts:

A decision in a petition for naturalization becomes final only


after 30 days from its promulgation and, insofar as the Solicitor
General is concerned, that period is counted from the date of his
receipt of the copy of the decision (Republic v. Court of First
Instance of Albay, 60 SCRA 195 [1974]).
Section 1 of R.A. No. 530 provides that no decision granting
citizenship in naturalization proceedings shall be executory until
after two years from its promulgation in order to be able to
observe if: (1) the applicant has left the country; (2) the
applicant has dedicated himself continuously to a lawful calling
or profession; (3) the applicant has not been convicted of any
offense or violation of government promulgated rules; and (4)
the applicant has committed any act prejudicial to the interest
of the country or contrary to government announced policies.
Even discounting the provisions of R.A. No. 530, the courts
cannot implement any decision granting the petition for
naturalization before its finality.
In view of the above findings, the supreme court ruled that
Frivaldo is declared NOT a citizen of the Philippines and
therefore disqualified from continuing to serve as governor of
the Province of Sorsogon. He is ordered to vacate his office and
to surrender the same to the Vice-Governor of the Province of
Sorsogon once this decision becomes final and executory.
Just in case i-ask:
The Comelec dismissed the petition of the State to declare the
proclamation of Frivaldo null and void for failure to appeal within
the 10-day period for appeal prescribed in Section 253 of the
Omnibus Election Code.
Ruling: The COMELEC failed to resolve the more serious issue
the disqualification of private respondent to be proclaimed
Governor on grounds of lack of Filipino citizenship. In this aspect,
the petition is one for quo warranto. In Frivaldo v. Commission
on Elections, 174 SCRA 245 (1989), we held that a petition
for quo warranto, questioning the respondents title and seeking
to prevent him from holding office as Governor for alienage, is
not covered by the ten-day period for appeal prescribed in
Section 253 of the Omnibus Election Code. Furthermore, we
explained that "qualifications for public office are continuing
requirements and must be possessed not only at the time of
appointment or election or assumption of office but during the
officers entire tenure; once any of the required qualification is
lost, his title may be seasonably challenge.
Labo vs. COMELEC (1989)
RAMON
L.
LABO,
JR.,
vs.THE COMMISSION ON ELECTIONS (COMELEC) EN BANC
AND LUIS L. LARDIZABAL

Ramon Labo was proclaimed mayor-elect of Baguio City, on


January 20, 1988.
A petition for quo warranto was filed by Laridzabal on
January 26, 1988.
Prior to this petition for quo warranto, two administrative
decisions had already been rendered on the question of the
Labos citizenship. The first was rendered by the
Commission on Elections on May 12, 1982, and foundLabo
to be a citizen of the Philippines. The second was rendered
by the Commission on Immigration and Deportation on
September 13, 1988, and held that the Labo was not a
citizen of the Philippines.
In the first case, it was noted that no direct evidence had
been presented to COMELEC to prove that Labo had been
naturalized as a citizen of Australia.
On the other hand, the decision of the CID took into account
the official statement of the Australian Government dated
August 12, 1984, through its Consul in the Philippines, that
the Labo was still an Australian citizen as of that date by
reason of his naturalization in 1976.
The facts then, with regard to Labos situation are: he had
married an Australian citizen, obtained an Australian
passport, and registered as an alien with the CID upon his
return to this country in 1980.
Labo claims that his naturalization in Australia made him a
dual citizen and did not divest him of his citizenship.
One of the issues in this case was whether the petition was
filed on time. Court ruled in the affirmative, pointing out
that the quo warranto proceeding had initially begun as a
pre-proclamation proceeding hence did not require the
payment of docket fees for perfection.

Issue: Whether or not Labo is a Filipino citizen as required by the


qualifications of his office. NO
Held:
C.A. 63 enumerates
citizenship is lost:

modes

by

which

PH

Such a specious argument cannot stand against the clear


provisions of CA No. 63, which enumerates the modes by which
Philippine citizenship may be lost. Among these are: (1)
naturalization in a foreign country; (2) express renunciation of
citizenship; and (3) subscribing to an oath of allegiance to
support the Constitution or laws of a foreign country, all of which
are applicable to the petitioner.
Annulment of naturalization under Australian law did not
restore PH citizenship
Even if it be assumed that, as the petitioner asserts, his
naturalization in Australia was annulled after it was found that
his marriage to the Australian citizen was bigamous, that
circumstance alone did not automatically restore his Philippine
citizenship. The possibility that he may have been subsequently
rejected by Australia, as he claims, does not mean that he has
been automatically reinstated as a citizen of the Philippines.
Means to reacquire citizenship: direct act of Congress, by
naturalization or repatriation

CO N F L I C T O F L AW S
N a t i o n a l i t y & D o m i c i l i a r y T h e o r y | 25
Under CA No. 63 as amended by PD No. 725, Philippine
citizenship may be reacquired by direct act of Congress, by
naturalization, or by repatriation. It does not appear in the
record, nor does the petitioner claim, that he has reacquired
Philippine citizenship by any of these methods. He does not
point to any judicial decree of naturalization as to any statute
directly conferring Philippine citizenship upon him. Neither has
he shown that he has complied with PD No. 725, providing that:
... (2) natural-born Filipinos who have lost their Philippine
citizenship may reacquire Philippine citizenship through
repatriation by applying with the Special Committee on
Naturalization created by Letter of Instruction No. 270, and, if
their applications are approved, taking the necessary oath of
allegiance to the Republic of the Philippines, after which they
shall be deemed to have reacquired Philippine citizenship. The
Commission on Immigration and Deportation shall thereupon
cancel their certificate of registration.
In case Sir asks about Labos overt
demonstrated his rejection of PH citizenship:

acts

To prove that Labo is not a Filipino citizen, Ortega submitted


the decision of the Supreme Court in "Ramon L. Labo, Jr.,
petitioner, v. Comelec, et al.," GR No. 86564, August 1,
1989, which declared Labo as NOT a citizen of the
Philippines and therefore DISQUALIFIED from continuing to
serve as Mayor of Baguio City. He is ordered to VACATE his
office and surrender the same to the Vice-Mayor of Baguio
City once this decision becomes final and executory.

On May 15, 1992, petitioner Labo filed the instant petition


for review docketed as G.R. No. 105111 with prayer, among
others, for the issuance of a temporary restraining order to
set aside the resolution of COMELEC to render judgment
declaring him as a Filipino citizen; and to direct COMELEC to
proceed with his proclamation in the event he wins in the
contested elections.

that

The petitioner does not question the authenticity of the above


evidence(referring to official statement from Australian
Government.) Neither does he deny that he obtained Australian
Passport No. 754705, which he used in coming back to the
Philippines in 1980, when he declared before the immigration
authorities that he was an alien and registered as such under
Alien Certificate of Registration No. B-323985. He later asked for
the change of his status from immigrant to a returning former
Philippine citizen and was granted Immigrant Certificate of
Residence No. 223809. He also categorically declared that he
was a citizen of Australia in a number of sworn statements
voluntarily made by him and. even sought to avoid the
jurisdiction of the barangay court on the ground that he was a
foreigner.
LABO v COMELEC

ISSUE:

NO. Labo failed to show that he reacquired his Filipino


citizenship. No evidence was adduced for Labo as in fact he
had no Answer as of the hearing. Up to this moment, Labo
still failed to submit a scintilla of proof to shore his claim
before this Court that he has indeed reacquired his
Philippine citizenship.

Labo's status has not changed in the case at bar. To


reiterate, Labo was disqualified as a candidate for being an
alien. His election does not automatically restore his
Philippine citizenship, the possession of which is an
indispensable requirement for holding public office (Sec. 39,
Local Government Code).

On May 5, 1992, Labo filed his verified Answer, insisting


that he is a Filipino citizen and continue to maintain and
preserve his Filipino citizenship; that he does not hold an
Australian citizenship; that the doctrine of res judicata does
not apply in citizenship; and that "existing facts support his
continuous maintenance and holding of Philippine
citizenship" and "supervening events now preclude the
application of the ruling in the Labo v. Comelec case and
the respondent (Labo) now hold and enjoys Philippine
citizenship.

No evidence has been offered by Labo to show what these


existing facts and supervening events are to preclude the
application of the Labo decision.

FACTS:

For the second time, Ramon Labo, Jr., believing that he is a


Filipino citizen, filed his certificate of candidacy for mayor of
Baguio City on March 23, 1992 for the May 1992 elections.
On the other hand, Roberto Ortego also filed his COC for the
same office.
Ortega filed a disqualification proceeding against Labo
before the COMELEC on the ground that Labo is not a
Filipino citizen.
On May 9, 1992, COMELEC issued the assailed resolution
denying and cancelling Labos COC on the ground that he is
not a Filipino citizen.

Labo previously cited the 1980 US case of Vance v.


Terrazas, wherein it was held that in proving expatriation,
an expatriating act an intent to relinquish citizenship must
be proved by a preponderance of evidence.

Labo claimed, however, that Sec. 72 of the Omnibus


Election Code "operates as a legislatively mandated special
repatriation proceeding" and that it allows his proclamation
as the winning candidate since the resolution disqualifying
him was not yet final at the time the election was held.
Labo also claimed that he has already filed an application
for reacquisition of Philippine citizenship before the office of

WON Labo reacquired his Filipino citizenship to


become qualified to run for mayor and be
declared as the winner? NO.

HELD:

211 SCRA 297 July 3, 1992

the Solicitor General, which was favored by the Solicitor


General.
However, COMELEC resolved to suspend the proclamation
of Labo in the event he wins in the elections for the City
Mayor of Baguio.

CO N F L I C T O F L AW S
N a t i o n a l i t y & D o m i c i l i a r y T h e o r y | 26

COMELEC is bound by the final declaration that Labo is not


a Filipino citizen. Consequently, Labos verified statement in
his certificate of candidacy that he is a "natural-born"
Filipino citizen is a false material representation."

The resolution cancelling Labo's certificate of candidacy on


the ground that he is not a Filipino citizen having acquired
finality on May 14, 1992 constrains the SC to rule against
his proclamation as Mayor of Baguio City.

At any rate, the fact remains that Labo has not submitted in
the instant case any evidence, if there be any, to prove his
reacquisition of Philippine citizenship either before the
COMELEC or SC. The COMELEC committed no grave abuse
of discretion in cancelling Labo's certificate of candidacy
and declaring that he is NOT a Filipino citizen pursuant to
the SCs ruling in the 1989 case of Labo v. Comelec (supra).

Undoubtedly, Labo, not being a Filipino citizen, lacks the


fundamental qualification for the contested office. Philippine
citizenship is an indispensable requirement for holding an
elective office. As mandated by law: "An elective local
official must be a citizen of the Philippines."

The issue here is citizenship and/or Labo's alienage the


very essence which strikes at the very core of petitioner
Labo's qualification to assume the contested office, he
being an alien and not a Filipino citizen. The fact that he
was elected by the majority of the electorate is of no
moment.

As for Labos argument citing Sec. 72 of the Omnibus


Election Code. In the first place, Sec. 72 of the Omnibus
Election Code has already been repealed by Sec. 6 of RA
No. 6646, to wit:
Sec. 6. Effect of Disqualification Case. Any candidate who
has been declared by final judgment to be disqualified shall
not be voted for, and the votes cast for him shall not be
counted. If for any reason a candidate is not declared by
final judgment before an election to be disqualified and he
is voted for and receives the winning number of votes in
such election, the Court or the Commission shall continue
with the trial and hearing of the action, inquiry, or
protest and, upon motion of the complainant or any
intervenor, may during the pendency thereof order the
suspension of the proclamation of such candidate whenever
the evidence of his guilt is strong.

A perusal of the above provision would readily disclose that


the COMELEC can legally suspend the proclamation of Labo,
his reception of the winning number of votes
notwithstanding. Labo failed to present any evidence before
the COMELEC to support his claim of reacquisition of
Philippine citizenship.
Furthermore, we need only to reiterate what we have stated
in Labo v. Comelec (supra), viz.,:
Under CA No. 63, as amended by P.D. No. 725, Philippine
citizenship may be reacquired by a direct act of Congress,
by naturalization, or by repatriation. It does not appear in
the record, nor does the petitioner claim, that he has
reacquired Philippine citizenship by any of these methods.
He does not point to any judicial decree of naturalization or
to any statute directly conferring Philippine citizenship upon
him. . . .

Despite the favorable recommendation by the Solicitor


General on Labos application for reacquisition of Philippine
citizenship, the Special Committee on Naturalization had
not yet acted upon said application for repatriation. Indeed,
such fact is even admitted by Labo. In the absence of any
official action or approval by the proper authorities, a mere
application for repratriation, does not, and cannot, amount
to an automatic reacquisition of the applicant's Philippine
citizenship.

SC dismissed the petition.


G.R. No. 120295 June 28, 1996
JUAN
G.
FRIVALDO, petitioner, vs.COMMISSION
ELECTIONS, and RAUL R. LEE, respondents.

ON

Facts:(sorry taaskaauang case)

On March 20, 1995, private respondent Juan G. Frivaldo filed


his COC for the office of Governor of Sorsogon in the May 8,
1995 elections.

On March 23, 1995, petitioner Raul R. Lee, another


candidate, filed a petition with the Comelec praying that
Frivaldo "be disqualified by reason of not yet being a citizen
of the Philippines", and that his COC be canceled. Comelec
granted the petition

Frivaldofiled an MFR but remained unacted upon until after


the May 8, 1995 elections. So, his candidacy continued and
he was voted for during the elections held on said date.

In the election Frivaldo obtained the highest number of


votes. Lee obtained the second highest number of votes.

On June 9, 1995, Lee filed a petition praying for his


proclamation
as
the
duly-elected
Governor
of
Sorsogon.Accordingly, at 8:30 in the evening of June 30,
1995, Lee was proclaimed governor of Sorsogon.

On July 6, 1995, Frivaldo filed with the Comelec a new


petition, praying for the annulment of the proclamation of
Lee and for his own proclamation. He alleged that on June
30, 1995, at 2:00 in the afternoon, he took his oath of
allegiance as a citizen of the Philippines after "his petition
for repatriation under P.D. 725 which he filed with the
Special Committee on Naturalization in September 1994
had been granted". As such, when "the said order (dated
June 21, 1995) (of the Comelec) . . . was released and
received by Frivaldo on June 30, 1995 at 5:30 o'clock in the

CO N F L I C T O F L AW S
N a t i o n a l i t y & D o m i c i l i a r y T h e o r y | 27
evening, there was no more legal impediment to the
proclamation (of Frivaldo) as governor.
Petitioner Lees contentions:
1. The judicially declared disqualification of respondent was
a continuing condition and rendered him ineligible to run
for, to be elected to and to hold the Office of Governor;
2. The alleged repatriation of respondent was neither valid
nor is the effect thereof retroactive as to cure his
ineligibility and qualify him to hold the Office of Governor;
and
Issues:
1. WoN the repatriation of Frivaldo valid and legal? Yes
2. WoN judicially declared disqualification
Filipino citizenship is a continuing bar to
run for, be elected to or hold the position? No

for lack of
hiseligibility to

Ruling:
The First Issue: Frivaldo's Repatriation
The Local Government Code of 1991 expressly requires
Philippine citizenship as a qualification for elective local officials,
including that of provincial governor. Under Philippine
law, citizenship may be reacquired by direct act of Congress, by
naturalization or by repatriation
First,Lee also argues that the repatriation proceedings was
irregular asserting that Frivaldo's application therefor was "filed
on June 29, 1995 (and) was approved in just one day or on June
30, 1995", which "prevented a judicious review and evaluation
of the merits thereof."
The court ruled that the presumption of regularity in the
performance of official duty and the presumption of legality in
the repatriation of Frivaldo have not been successfully rebutted
by Lee. The mere fact that the proceedings were speeded up is
by itself not a ground to conclude that such proceedings were
necessarily tainted. After all, the requirements of repatriation
under P.D. No. 725 are not difficult to comply with, nor are they
tedious and cumbersome. This is not unusual since, unlike in
naturalization where an alien covets a first-timeentry into
Philippine political life, in repatriation the applicant is a former
natural-born Filipino who is merely seeking to reacquire his
previous citizenship. In the case of Frivaldo, he was undoubtedly
a natural-born citizen who openly and faithfully served his
country and his province prior to his naturalization in the United
States
Second, Lee further contends that assuming the assailed
repatriation to be valid, nevertheless it could only be effective
as at 2:00 p.m. of June 30, 1995 whereas the citizenship
qualification prescribed by the Local Government Code "must
exist on the date of his election, if not when the certificate of
candidacy is filed,"

However under sec. 29 of the LGC the law does not specify any
particular date or time when the candidate must possess
citizenship, unlike that for residence (which must consist of at
least one year's residency immediately preceding the day of
election) and age (at least twenty three years of age on election
day).
Philippine citizenship is an indispensable requirement for holding
an elective public office, and the purpose of the citizenship
qualification is none other than to ensure that no alien, shall
govern our people and our country or a unit of territory thereof.
Now, an official begins to govern or to discharge his functions
only upon his proclamation and on the day the law mandates his
term of office to begin. Since Frivaldo re-assumed his citizenship
on June 30, 1995 -- the very day the term of office of governor
(and other elective officials) began -- he was therefore already
qualified to be proclaimed, to hold such office and to discharge
the functions and responsibilities thereof as of said date. In
short, at that time, he was already qualified to govern his native
Sorsogon. This is the liberal interpretation that should give spirit,
life and meaning to our law on qualifications consistent with the
purpose for which such law was enacted.
But to remove all doubts on this important issue, we also hold
that the repatriation of Frivaldo RETROACTED to the date of the
filing of his application on August 17, 1994.
It is true that under the Civil Code of the Philippines, "(l)aws
shall have no retroactive effect, unless the contrary is provided."
But there are settled exceptions to this general rule, such as
when the statute is CURATIVE or REMEDIAL in nature or when it
CREATES NEW RIGHTS.
Being a former Filipino who has served the people repeatedly,
Frivaldo deserves a liberal interpretation of Philippine laws and
whatever defects there were in his nationality should now be
deemed mooted by his repatriation.
It is not disputed that on January 20, 1983 Frivaldo became an
American. Would the retroactivity of his repatriation not
effectively give him dual citizenship, which under Sec. 40 of the
Local Government Code would disqualify him "from running for
any elective local position?"
We answer this question in the negative, as there is cogent
reason to hold that Frivaldo was really STATELESS at the time he
took said oath of allegiance and even before that, when he ran
for governor in 1988. In his Comment, Frivaldo wrote that he
"had long renounced and had long abandoned his American
citizenship -- long before May 8, 1995. At best, Frivaldo was
stateless in the interim -- when he abandoned and renounced
his US citizenship but before he was repatriated to his Filipino
citizenship."
On this point, we quote from the assailed Resolution dated
December 19, 1995:
By the laws of the United States, petitioner Frivaldo lost his
American citizenship when he took his oath of allegiance to
the Philippine Government when he ran for Governor in
1988, in 1992, and in 1995. Every certificate of candidacy
contains an oath of allegiance to the Philippine Government."

CO N F L I C T O F L AW S
N a t i o n a l i t y & D o m i c i l i a r y T h e o r y | 28
The Second Issue:
Disqualification?

Is

Lack

of

Citizenshipa

Continuing

We do not agree.
Indeed, decisions declaring the acquisition or denial of
citizenship cannot govern a person's future status with finality.
This is because a person may subsequently reacquire, or for that
matter lose, his citizenship under any of the modes recognized
by law for the purpose. Hence, in Lee vs. Commissioner of
Immigration, we held:
Everytime the citizenship of a person is material or
indispensable in a judicial or administrative case, whatever the
corresponding court or administrative authority decides therein
as to such citizenship is generally not considered res judicata,
hence it has to be threshed out again and again, as the occasion
demands.

B.M. No. 1678

remind him of his duties and responsibilities as a member of the


Philippine bar.
ISSUE: How is Filipino citizenship reacquired? (Main issue in this
case: Given the foregoing, may a lawyer who has lost his Filipino
citizenship still practice law in the Philippines? No.)
HELD: The Constitution provides that the practice of all
professions in the Philippines shall be limited to Filipino citizens
save in cases prescribed by law. Since Filipino citizenship is a
requirement for admission to the bar, loss thereof terminates
membership in the Philippine bar and, consequently, the
privilege to engage in the practice of law. In other words, the
loss of Filipino citizenship ipso jure terminates the privilege to
practice law in the Philippines. The practice of law is a privilege
denied to foreigners.
The exception is when Filipino citizenship is lost by reason of
naturalization as a citizen of another country but subsequently
reacquired pursuant to RA 9225. This is because "all Philippine
citizens who become citizens of another country shall
be deemed not to have lost their Philippine citizenship under the
conditions of [RA 9225]." Therefore, a Filipino lawyer who
becomes a citizen of another country is deemed never to have
lost his Philippine citizenship if he reacquires it in
accordance with RA 9225. Although he is also deemed never
to have terminated his membership in the Philippine bar, no
automatic right to resume law practice accrues.

December 17, 2007

PETITION FOR LEAVE TO RESUME PRACTICE OF LAW,


BENJAMIN M. DACANAY, petitioner.
FACTS: Petitioner was admitted to the Philippine bar in March
1960. He practiced law until he migrated to Canada in
December 1998 to seek medical attention for his ailments. He
subsequently applied for Canadian citizenship to avail of
Canadas free medical aid program. His application was
approved and he became a Canadian citizen in May 2004.
On July 14, 2006, pursuant to Republic Act (RA) 9225
(Citizenship Retention and Re-Acquisition Act of 2003), petitioner
reacquired his Philippine citizenship.1 On that day, he took his
oath of allegiance as a Filipino citizen before the Philippine
Consulate General in Toronto, Canada. Thereafter, he returned to
the Philippines and now intends to resume his law practice.
There is a question, however, whether petitioner Benjamin M.
Dacanay lost his membership in the Philippine bar when he gave
up his Philippine citizenship in May 2004. Thus, this petition.
In a report dated October 16, 2007, the Office of the Bar
Confidant cites Section 2, Rule 138 (Attorneys and Admission to
Bar) of the Rules of Court:
SECTION 2. Requirements for all applicants for admission to the
bar. Every applicant for admission as a member of the
bar must be a citizen of the Philippines, at least twenty-one
years of age, of good moral character, and a resident of the
Philippines; and must produce before the Supreme Court
satisfactory evidence of good moral character, and that no
charges against him, involving moral turpitude, have been filed
or are pending in any court in the Philippines.
Applying the provision, the Office of the Bar Confidant opines
that, by virtue of his reacquisition of Philippine citizenship, in
2006, petitioner has again met all the qualifications and has
none of the disqualifications for membership in the bar. It
recommends that he be allowed to resume the practice of law in
the Philippines, conditioned on his retaking the lawyers oath to

Under RA 9225, if a person intends to practice the legal


profession in the Philippines and he reacquires his Filipino
citizenship pursuant to its provisions "(he) shall apply with the
proper authority for a license or permit to engage in such
practice." Stated otherwise, before a lawyer who reacquires
Filipino citizenship pursuant to RA 9225 can resume his law
practice, he must first secure from this Court the authority to do
so, conditioned on:
(a) the updating and payment in full of the annual membership
dues in the IBP;
(b) the payment of professional tax;
(c) the completion of at least 36 credit hours of mandatory
continuing legal education; this is specially significant to refresh
the applicant/petitioners knowledge of Philippine laws and
update him of legal developments and
(d) the retaking of the lawyers oath which will not only
remind him of his duties and responsibilities as a lawyer and as
an officer of the Court, but also renew his pledge to maintain
allegiance to the Republic of the Philippines.
Compliance with these conditions will restore his good standing
as a member of the Philippine bar.

CO N F L I C T O F L AW S
N a t i o n a l i t y & D o m i c i l i a r y T h e o r y | 29
Urgent Motion Ad Cautelum to Suspend Proclamation of
petitioner, which the COMELEC en banc granted.
COMELEC en banc then declared Aquino ineligible and thus
disqualified as a candidate for the Office of Representative of
the Second Legislative District of Makati City in the May 8, 1995
elections, for lack of the constitutional qualification of residence.
Hence, the instant Petition for Certiorari.
Issue:
WON Aquino is deemed a resident of Makati to qualify
as candidate for the position of Representative.
Held:
No.
The Constitution requires that a person
seeking election to the House of Representatives should be a
resident of the district in which he seeks election for a period of
not less than one (l) year prior to the elections. Residence, for
election law purposes, has a settled meaning in our jurisdiction.
The term "residence" has always been understood as
synonymous with "domicile" not only under the previous
Constitutions but also under the 1987 Constitution (Co v.
Electoral Tribunal of the House of Representatives). The framers
of the Constitution adhered to the earlier definition given to the
word "residence" which regarded it as having the same meaning
as domicile.Clearly, the place "where a party actually or
constructively has his permanent home," where he, no matter
where he may be found at any given time, eventually intends to
return and remain, i.e., his domicile, is that to which the
Constitution refers when it speaks of residence for the purposes
of election law.

IX. DOMICILIARY THEORY


G.R. No. 120265 September 18, 1995
AGAPITO A. AQUINO vs.COMMISSION ON ELECTIONS,
MOVE MAKATI, MATEO BEDON and JUANITO ICARO
Facts:
On March 20, 1995, Aquino filed his Certificate of
Candidacy (COC) for the position of Representative for the new
Second Legislative District of Makati City. Among others, Aquino
provided in his COC that he is a resident of 284 AMAPOLA
COR. ADALLA STS., PALM VILLAGE, MAKATI.
Move Makati (political party) and Mateo Bedon, Chairman of the
LAKAS-NUCD-UMDP filed a petition to disqualify Aquino on the
ground that the latter lacked the residence qualification as a
candidate for congressman which, under Section 6, Art. VI of the
1987 the Constitution, should be for a period not less than one
(1) year immediately preceding the May 8, 1995 elections.
One day after said petition for disqualification was filed,
petitioner amended his COC, stating that he had resided in the
constituency where he sought to be elected for one (l) year and
thirteen (13) days. A hearing was conducted by the COMELEC
wherein petitioner testified and presented in evidence, among
others, his Affidavit dated May 2, 1995, lease contract between
petitioner and Leonor Feliciano dated April 1, 1994, Affidavit of
Leonor Feliciano dated April 28,1995 and Affidavit of
DanielGalamay dated April 28, 1995.
After hearing of the petition for disqualification, the Second
Division of the COMELEC promulgated a Resolution dismissing
the Petition for Disualification. Pending the MR before the
COMELEC en banc,elections were held in which Aquino garnered
the highest number of votes. Private respondents then filed an

The manifest purpose of this deviation from the usual


conceptions of residency in law as explained in Gallego vs. Vera
at is "to exclude strangers or newcomers unfamiliar with the
conditions and needs of the community" from taking advantage
of favorable circumstances existing in that community for
electoral gain. While there is nothing wrong with the practice of
establishing residence in a given area for meeting election law
requirements, this nonetheless defeats the essence of
representation, which is to place through the assent of voters
those most cognizant and sensitive to the needs of a particular
district, if a candidate falls short of the period of residency
mandated by law for him to qualify. That purpose could be
obviously best met by individuals who have either had actual
residence in the area for a given period or who have been
domiciled in the same area either by origin or by choice. It
would, therefore, be imperative for this Court to inquire into the
threshold question as to whether or not petitioner actually was a
resident for a period of one year in the area now encompassed
by the Second Legislative District of Makati at the time of his
election or whether or not he was domiciled in the same.
In this case, the COMELEC en banc found that Aquinos domicile
of origin of record up to the time of filing of his most recent
certificate of candidacy for the 1995 elections was Concepcion,
Tarlac.Petitioner's alleged connection with the Second District
of Makati City is an alleged lease agreement of condominium
unit in the area. According to the COMELEC, the intention not to
establish a permanent home in Makati City is evident in his
leasing a condominium unit instead of buying one. While a lease
contract maybe indicative of respondent's intention to reside in
Makati City it does not engender the kind of permanency
required to prove abandonment of one's original domicile
especially since, by its terms, it is only for a period of two (2)
years, and respondent Aquino himself testified that his intention
was really for only one (l) year because he has other
"residences" in Manila or Quezon City.
While property ownership is not and should never be an indicia
of the right to vote or to be voted upon, the fact that petitioner

CO N F L I C T O F L AW S
N a t i o n a l i t y & D o m i c i l i a r y T h e o r y | 30
himself claims that he has other residences in Metro Manila
coupled with the short length of time he claims to be a resident
of the condominium unit in Makati (and the fact, of his stated
domicile in Tarlac) "indicate that the sole purpose of (petitioner)
in transferring his physical residence" is not to acquire's new
residence or domicile "but only to qualify as a candidate for
Representative of the Second District of Makati City." The
absence of clear and positive proof showing a successful
abandonment of domicile under the conditions stated above, the
lack of identification sentimental, actual or otherwise with
the area, and the suspicious circumstances under which the
lease agreement was effected all belie petitioner's claim of
residency for the period required by the Constitution, in the
Second District of Makati.
Moreover, his assertion that he has transferred his domicile from
Tarlac to Makati is a bare assertion which is hardly supported by
the facts in the case at bench. Domicile of origin is not easily
lost. To successfully effect a change of domicile, petitioner must
prove an actual removal or an actual change of domicile; a bona
fide intention of abandoning the former place of residence and
establishing a new one and definite acts which correspond with
the purpose These requirements are hardly met by the evidence
adduced in support of petitioner's claims of a change of domicile
from Tarlac to the Second District of Makati. In the absence of
clear and positive proof, the domicile of origin be deemed to
continue requirements are hardly met by the evidence adduced
in support of petitioner's claims of a change of domicile from
Tarlac to the Second District of Makati. In the absence of clear
and positive proof, the domicile of origin should be deemed to
continue.
Hence, the instant petition was DISMISSED.

IMELDA ROMUALDEZ-MARCOS, petitioner,


and CIRILO ROY MONTEJO, respondents.

vs.

COMELEC

FACTS:

Petitioner Imelda Romualdez-Marcos filed her Certificate of


Candidacy for the position of Representative of the First
District of Leyte with the Provincial Election Supervisor on
March 8, 1995, providing the following information in item
no. 8:
RESIDENCE IN THE CONSTITUENCY WHERE I SEEK TO BE
ELECTED
IMMEDIATELY
PRECEDING
THE
ELECTION:
__________ Years and seven Months.

On March 23, 1995, private respondent Cirilo Roy Montejo,


the incumbent Representative of the First District of Leyte
and a candidate for the same position, filed a "Petition for
Cancellation and Disqualification" with the Commission on
Elections alleging that petitioner did not meet the
constitutional requirement for residency.

In his petition, private respondent contended that Mrs.


Marcos lacked the Constitution's one year residency

requirement
for
candidates
for
the
House
of
Representatives on the evidence of declarations made by
her in Voter Registration Record 94-No. 3349772 and in her
Certificate of Candidacy. He prayed that "an order be issued
declaring (petitioner) disqualified and canceling the
certificate of candidacy."

Marcos tried to correct/amend her COC by placing since


childhood referring to her residency.

The amended COC was not accepted and Marcos was later
declared as not qualified to run for the position of Member
of the House of Representative.

Marcos won, overwhelmingly, the election and COMELEC


suspended her proclamation.

She then went to the SC to question her disqualification on


the basis of non-fulfillment of the residency requirement.

ISSUE: Whether or not petitioner was a resident, for


election purposes, of the First District of Leyte for a
period of one year at the time of the May 9, 1995
elections.
RULING:
A perusal of the Resolution of the COMELEC's
Second Division reveals a startling confusion in the application
of settled concepts of "Domicile" and "Residence" in election
law.
Residence, in its ordinary conception, implies the factual
relationship of an individual to a certain place. It is the physical
presence of a person in a given area, community or country. The
essential distinction between residence and domicile in law is
that residence involves the intent to leave when the purpose for
which the resident has taken up his abode ends. One may seek
a place for purposes such as pleasure, business, or health. If a
person's intent be to remain, it becomes his domicile; if his
intent is to leave as soon as his purpose is established it is
residence. It is thus, quite perfectly normal for an individual to
have different residences in various places. However, a person
can only have a single domicile, unless, for various reasons, he
successfully abandons his domicile in favor of another domicile
of choice. In Uytengsu vs. Republic, we laid this distinction quite
clearly:
There is a difference between domicile and residence.
"Residence" is used to indicate a place of abode,
whether permanent or temporary; "domicile" denotes a
fixed permanent residence to which, when absent, one
has the intention of returning. A man may have a
residence in one place and a domicile in another.
Residence is not domicile, but domicile is residence
coupled with the intention to remain for an unlimited
time. A man can have but one domicile for the same
purpose at any time, but he may have numerous places
of residence. His place of residence is generally his
place of domicile, but it is not by any means
necessarily so since no length of residence without
intention of remaining will constitute domicile.

CO N F L I C T O F L AW S
N a t i o n a l i t y & D o m i c i l i a r y T h e o r y | 31
For political purposes the concepts of residence and domicile are
dictated by the peculiar criteria of political laws. As these
concepts have evolved in our election law, what has clearly and
unequivocally emerged is the fact that residence for election
purposes is used synonymously with domicile.
In Nuval vs. Guray, the Court held that "the term residence. . . is
synonymous with domicile which imports not only intention to
reside in a fixed place, but also personal presence in that place,
coupled with conduct indicative of such intention."
In the light of the principles just discussed, has petitioner Imelda
Romualdez Marcos satisfied the residency requirement
mandated by Article VI, Sec. 6 of the 1987 Constitution? Of what
significance is the questioned entry in petitioner's Certificate of
Candidacy stating her residence in the First Legislative District
of Leyte as seven (7) months?
It is the fact of residence, not a statement in a certificate of
candidacy which ought to be decisive in determining whether or
not and individual has satisfied the constitution's residency
qualification requirement. The said statement becomes material
only when there is or appears to be a deliberate attempt to
mislead, misinform, or hide a fact which would otherwise render
a candidate ineligible. It would be plainly ridiculous for a
candidate to deliberately and knowingly make a statement in a
certificate of candidacy which would lead to his or her
disqualification.
It stands to reason therefore, that petitioner merely committed
an honest mistake in jotting the word "seven" in the space
provided for the residency qualification requirement. The
circumstances leading to her filing the questioned entry
obviously resulted in the subsequent confusion which prompted
petitioner to write down the period of her actual stay in Tolosa,
Leyte instead of her period of residence in the First district,
which was "since childhood" in the space provided.
We now proceed to the matter of petitioner's domicile.
In support of its asseveration that petitioner's domicile could not
possibly be in the First District of Leyte, the Second Division of
the COMELEC, in its assailed Resolution of April 24,1995
maintains that "except for the time when (petitioner) studied
and worked for some years after graduation in Tacloban City,
she continuously lived in Manila." The Resolution additionally
cites certain facts as indicative of the fact that petitioner's
domicile ought to be any place where she lived in the last few
decades except Tacloban, Leyte.
First, according to the Resolution, petitioner, in 1959, resided in
San Juan, Metro Manila where she was also registered voter.
Then, in 1965, following the election of her husband to the
Philippine presidency, she lived in San Miguel, Manila where she
as a voter. In 1978 and thereafter, she served as a member of
the Batasang Pambansa and Governor of Metro Manila. "She
could not, have served these positions if she had not been a
resident of Metro Manila," the COMELEC stressed. Here is where
the confusion lies.
We have stated, many times in the past, that an individual does
not lose his domicile even if he has lived and maintained

residences in different places. Residence, it bears repeating,


implies a factual relationship to a given place for various
purposes. The absence from legal residence or domicile to
pursue a profession, to study or to do other things of a
temporary or semi-permanent nature does not constitute loss of
residence. Thus, the assertion by the COMELEC that "she could
not have been a resident of Tacloban City since childhood up to
the time she filed her certificate of candidacy because she
became a resident of many places" flies in the face of settled
jurisprudence in which this Court carefully made distinctions
between (actual) residence and domicile for election law
purposes. In Larena vs. Teves, supra, we stressed:
[T]his court is of the opinion and so holds that a person
who has his own house wherein he lives with his family
in a municipality without having ever had the intention
of abandoning it, and without having lived either alone
or with his family in another municipality, has his
residence in the former municipality, notwithstanding
his having registered as an elector in the other
municipality in question and having been a candidate
for various insular and provincial positions, stating
every time that he is a resident of the latter
municipality.
More significantly, in Faypon vs. Quirino, We explained that:
A citizen may leave the place of his birth to look for
"greener pastures," as the saying goes, to improve his
lot, and that, of course includes study in other places,
practice of his avocation, or engaging in business.
When an election is to be held, the citizen who left his
birthplace to improve his lot may desire to return to his
native town to cast his ballot but for professional or
business reasons, or for any other reason, he may not
absent himself from his professional or business
activities; so there he registers himself as voter as he
has the qualifications to be one and is not willing to
give up or lose the opportunity to choose the officials
who are to run the government especially in national
elections. Despite such registration, the animus
revertendi to his home, to his domicile or residence of
origin has not forsaken him. This may be the
explanation why the registration of a voter in a place
other than his residence of origin has not been deemed
sufficient to constitute abandonment or loss of such
residence. It finds justification in the natural desire and
longing of every person to return to his place of birth.
This strong feeling of attachment to the place of one's
birth must be overcome by positive proof of
abandonment for another.
From the foregoing, it can be concluded that in its above-cited
statements supporting its proposition that petitioner was
ineligible to run for the position of Representative of the First
District of Leyte, the COMELEC was obviously referring to
petitioner's various places of (actual) residence, not her
domicile. In doing so, it not only ignored settled jurisprudence
on residence in election law and the deliberations of the
constitutional commission but also the provisions of the
Omnibus Election Code (B.P. 881).

CO N F L I C T O F L AW S
N a t i o n a l i t y & D o m i c i l i a r y T h e o r y | 32
Applying the principles discussed to the facts found by
COMELEC, what is inescapable is that petitioner held various
residences for different purposes during the last four decades.
None of these purposes unequivocally point to an intention to
abandon her domicile of origin in Tacloban, Leyte. Moreover,
while petitioner was born in Manila, as a minor she naturally
followed the domicile of her parents.
She grew up in Tacloban, reached her adulthood there and
eventually established residence in different parts of the country
for various reasons. Even during her husband's presidency, at
the height of the Marcos Regime's powers, petitioner kept her
close ties to her domicile of origin by establishing residences in
Tacloban, celebrating her birthdays and other important
personal milestones in her home province, instituting wellpublicized projects for the benefit of her province and
hometown, and establishing a political power base where her
siblings and close relatives held positions of power either
through the ballot or by appointment, always with either her
influence or consent. These well-publicized ties to her domicile
of origin are part of the history and lore of the quarter century of
Marcos power in our country. Either they were entirely ignored in
the COMELEC'S Resolutions, or the majority of the COMELEC did
not know what the rest of the country always knew: the fact of
petitioner's domicile in Tacloban, Leyte.
Private respondent in his Comment, contends that Tacloban was
not petitioner's domicile of origin because she did not live there
until she was eight years old. He avers that after leaving the
place in 1952, she "abandoned her residency (sic) therein for
many years and . . . (could not) re-establish her domicile in said
place by merely expressing her intention to live there again." We
do not agree.
First, minor follows the domicile of his parents. As domicile, once
acquired is retained until a new one is gained, it follows that in
spite of the fact of petitioner's being born in Manila, Tacloban,
Leyte was her domicile of origin by operation of law. This
domicile was not established only when her father brought his
family back to Leyte contrary to private respondent's
averments.
Second, domicile of origin is not easily lost. To successfully effect
a change of domicile, one must demonstrate:
1. An actual removal or an actual change of domicile; 2. A bona
fide intention of abandoning the former place of residence and
establishing a new one; and 3. Acts which correspond with the
purpose.
In the absence of clear and positive proof based on these
criteria, the residence of origin should be deemed to continue.
Only with evidence showing concurrence of all three
requirements can the presumption of continuity or residence be
rebutted, for a change of residence requires an actual and
deliberate abandonment, and one cannot have two legal
residences at the same time. In the case at bench, the evidence
adduced by private respondent plainly lacks the degree of
persuasiveness required to convince this court that an
abandonment of domicile of origin in favor of a domicile of
choice indeed occurred. To effect an abandonment requires the
voluntary act of relinquishing petitioner's former domicile with

an intent to supplant the former domicile with one of her own


choosing (domicilium voluntarium).
Even assuming for the sake of argument that petitioner gained a
new "domicile" after her marriage and only acquired a right to
choose a new one after her husband died, petitioner's acts
following her return to the country clearly indicate that she not
only impliedly but expressly chose her domicile of origin
(assuming this was lost by operation of law) as her domicile.
This "choice" was unequivocally expressed in her letters to the
Chairman of the PCGG when petitioner sought the PCGG's
permission to "rehabilitate (our) ancestral house in Tacloban and
Farm in Olot, Leyte. . . to make them livable for the Marcos
family to have a home in our homeland."
Moreover, and proceeding from our discussion pointing out
specific situations where the female spouse either reverts to her
domicile of origin or chooses a new one during the subsistence
of the marriage, it would be highly illogical for us to assume that
she cannot regain her original domicile upon the death of her
husband absent a positive act of selecting a new one where
situations exist within the subsistence of the marriage itself
where the wife gains a domicile different from her husband.
In the light of all the principles relating to residence and
domicile enunciated by this court up to this point, we are
persuaded that the facts established by the parties weigh
heavily in favor of a conclusion supporting petitioner's claim of
legal residence or domicile in the First District of Leyte.

DOMINO v COMELEC
(ps.
This
is
a
digest
from
http://cofferette.blogspot.com/2009/01/domino-vs-comelec-grno-134015-july-19.html - Wala nagpasa si Madam Joan)
Facts: Petitioner Domino filed his certificate of candidacy for
the position of Representative of the lone legislative district of
the Province of Sarangani indicating that he has resided in the
constituency where he seeks to be elected for 1 year and 2
months. Private respondents filed a petition seeking to cancel
the certificate of candidacy of Domino, alleging that Domino,
contrary to his declaration in the certificate of candidacy, is not
a resident, much less a registered voter, of the province of
Sarangani where he seeks election. Thereafter, the COMELEC
promulgated
a
resolution
declaring
Domino disqualified as candidate for
the
position
of
representative of the lone district of Sarangani in the May 11,
1998 polls for lack of the one-year residency requirement and
likewise ordered the cancellation of his certificate of candidacy
based on his own Voters Registration Record and his
address indicated as 24 Bonifacio St., Ayala Hts., Old Balara,
Quezon
City.
Issue: Whether or not petitioner has resided in Sarangani
Province for at least 1 year immediately preceding the May 11,
1998
elections
Held: The term residence, as used in the law prescribing the
qualifications for suffrage and for elective office, means the
same thing as domicile, which imports not only an intention to
reside in a fixed place but also personal presence in that place,

CO N F L I C T O F L AW S
N a t i o n a l i t y & D o m i c i l i a r y T h e o r y | 33
coupled with conduct indicative of suchintention. Domicile
denotes a fixed permanent residence to which, whenever absent
for business, pleasure, or some other reasons, one intends to
return.
Records show that petitioners domicile of origin was Candon,
Ilocos Sur and that sometime in 1991, he acquired a new
domicile of choice in Quezon City, as shown by his certificate of
candidacy for the position of representative of the Third District
of Quezon City in the May 1995 election. Petitioner is now
claiming that he had effectively abandoned his residence in
Quezon City and has established a new domicile of choice in the
Province
of
Sarangani.
A persons domicile, once established, is considered to continue
and will not be deemed lost until a new one is established. To
successfully effect a change of domicile, one must demonstrate
an actual removal or an actual change of domicile; a bona
fide intention of abandoning the former place of residence and
establishing a new one and definite acts which correspond with
the
purpose.
The contract of lease of a house and lot entered into sometime
in January 1997 does not adequately support a change of
domicile.
The
lease
contract
may
be indicative of
Dominos intention to reside in Sarangani, but it does not
engender the kind of permanency required to prove
abandonment of ones original domicile. The mere absence of
individual from his permanent residence, no matter how long,
without the intention to abandon it does not result in loss or
change of domicile. Thus, the date of the contract of lease of
a house and lot in Sarangani cannot be used, in the absence of
other circumstances, as the reckoning period of the one-year
residence requirement. Further, Dominos lack of intention to
abandon his residence in Quezon City is strengthened by his act
of registering as voter in Quezon City. While voting is not
conclusive of residence, it does give rise to a strong
presumption of residence especially in this case where
Domino registered in his former barangay.
Jao v CA
FACTS:

Sps Jao died intestate. They left several properties to their


sons Rodolfo and Perico Jao.
Perico instituted a petition for the issuance of letters of
administration over the estate of his parents in the RTC of
Quezon City.
Rodolfo moved for the dismissal of the petition on the
ground of improper venue.
He alleged that the deceased spouses are not residents of
Quezon City either during their lifetime or at the time of
their deaths.
The decedents actual residence was in Angeles City,
Pampanga, where his late mother used to run and operate a
bakery.
As the health of his parents deteriorated due to old age,
they stayed in Rodolfos residence at Quezon City, solely for
the purpose of obtaining medical treatment and
hospitalization.
Rodolfo submitted documentary evidence previously
executed by the decedents, consisting of income tax
returns, voters affidavits, statements of assets and
liabilities, real estate tax payments, motor vehicle
registration and passports, all indicating that their
permanent residence was in Angeles City, Pampanga.
Perico submitted in evidence the death certificates of their
deceased parents showing that their last residence was in
Quezon City and that the same was filled out by Rodolfo.
Rodolfo alleged that he supplied the information in good
faith and through honest mistake. He gave his residence

only as reference and that their stay there was merely


transitory.
RTC and CA denied Rodolfos motion to dismiss.

ISSUE: Where should the settlement proceedings be had --- in


Pampanga, where the decedents had their permanent residence,
or in Quezon City, where they actually stayed before their
demise? It must be in QUEZON CITY.
HELD:

Rule 73, Section 1 of the Rules of Court states:

Where estate of deceased persons be settled. If the decedent


is an inhabitant of the Philippines at the time of his death,
whether a citizen or an alien, his will shall be proved, or letters
of administration granted, and his estate settled, in the Court of
First Instance in the province in which he resides at the time of
his death, and if he is an inhabitant of a foreign country, the
Court of First Instance of any province in which he had estate.
The court first taking cognizance of the settlement of the estate
of a decedent shall exercise jurisdiction to the exclusion of all
other courts. The jurisdiction assumed by a court, so far as it
depends on the place of residence of the decedent, or of the
location of his estate, shall not be contested in a suit or
proceeding, except in an appeal from that court, in the original
case, or when the want of jurisdiction appears on the record.
(underscoring ours)
Clearly, the estate of an inhabitant of the Philippines shall be
settled or letters of administration granted in the proper court
located in the province where the decedent resides at the time
of his death.
Entry in the death certificate is substantial proof of
residence
In the case at bar, there is substantial proof that the decedents
have transferred to petitioners Quezon City residence.
Petitioner failed to sufficiently refute respondents assertion that
their elderly parents stayed in his house for some three to four
years before they died in the late 1980s.
Furthermore, the decedents respective death certificates state
that they were both residents of Quezon City at the time of their
demise. Significantly, it was petitioner himself who filled up his
late mothers death certificate. To our mind, this unqualifiedly
shows that at that time, at least, petitioner recognized his
deceased mothers residence to be Quezon City. Moreover,
petitioner failed to contest the entry in Ignacios death
certificate, accomplished a year earlier by respondent.
The recitals in the death certificates, which are admissible in
evidence, were thus properly considered and presumed to be
correct by the court a quo. We agree with the appellate courts
observation that since the death certificates were accomplished
even before petitioner and respondent quarreled over their
inheritance, they may be relied upon to reflect the true situation
at the time of their parents death.
The death certificates thus prevailed as proofs of the decedents
residence at the time of death, over the numerous
documentary evidence presented by petitioner. To be sure, the
documents presented by petitioner pertained not to residence
at the time of death, as required by the Rules of Court, but
to permanent residence or domicile.
Term resides connotes ex vi termini "actual residence" as
distinguished from "legal residence or domicile." Even where the
statute uses the word "domicile" still it is construed as meaning
residence and not domicile in the technical sense. Some cases

CO N F L I C T O F L AW S
N a t i o n a l i t y & D o m i c i l i a r y T h e o r y | 34
make a distinction between the terms "residence" and
"domicile" but as generally used in statutes fixing venue, the
terms are synonymous, and convey the same meaning as the
term "inhabitant."
In other words, "resides" should be viewed or understood in its
popular sense, meaning, the personal, actual or physical
habitation of a person, actual residence or place of abode. It
signifies physical presence in a place and actual stay thereat. In
this popular sense, the term means merely residence, that is,
personal residence, not legal residence or domicile. Residence
simply requires bodily presence as an inhabitant in a given
place, while domicile requires bodily presence in that place and
also an intention to make it ones domicile. No particular length
of time of residence is required though; however, the residence
must be more than temporary
Both the settlement court and the Court of Appeals found that
the decedents have been living with petitioner at the time of
their deaths and for some time prior thereto. We find this
conclusion to be substantiated by the evidence on record. A
close perusal of the challenged decision shows that, contrary to
petitioners assertion, the court below considered not only the
decedents physical presence in Quezon City, but also other
factors indicating that the decedents stay therein was more
than temporary. In the absence of any substantial showing that
the lower courts factual findings stemmed from an erroneous
apprehension of the evidence presented, the same must be held
to be conclusive and binding upon this Court.
Petitioner strains to differentiate between the venue provisions
found in Rule 4, Section 2, on ordinary civil actions, and Rule 73,
Section 1, which applies specifically to settlement proceedings.
He argues that while venue in the former understandably refers
to actual physical residence for the purpose of serving
summons, it is the permanent residence of the decedent which
is significant in Rule 73, Section 1. Petitioner insists that venue
for the settlement of estates can only refer to permanent
residence or domicile because it is the place where the records
of the properties are kept and where most of the decedents
properties are located.
Petitioners argument fails to persuade.
It does not necessarily follow that the records of a persons
properties are kept in the place where he permanently resides.
Neither can it be presumed that a persons properties can be
found mostly in the place where he establishes his domicile. It
may be that he has his domicile in a place different from that
where he keeps his records, or where he maintains extensive
personal and business interests. No generalizations can thus be
formulated on the matter, as the question of where to keep
records or retain properties is entirely dependent upon an
individuals choice and peculiarities.
At any rate, petitioner is obviously splitting straws when he
differentiates between venue in ordinary civil actions and venue
in special proceedings. In Raymond v. Court of Appeals
and Bejer v. Court of Appeals, we ruled that venue for ordinary
civil actions and that for special proceedings have one and the
same meaning. As thus defined, "residence", in the context of
venue provisions, means nothing more than a persons actual
residence or place of abode, provided he resides therein with
continuity and consistency. All told, the lower court and the
Court of Appeals correctly held that venue for the settlement of
the decedents intestate estate was properly laid in the Quezon
City court.
Petition was denied.

PHILIP G. ROMUALDEZ, vs. RTC of TACLOBAN, DONATO


ADVINCULA, BOARD OF ELECTION INSPECTORS PRECINCT
No. 9, MALBOG, TOLOSA, LEYTE, and the MUNICIPAL
REGISTRAR COMELEC, TOLOSA, LEYTE. G.R. No. 104960,
September 14, 1993.

Philip Romualdez is a natural born citizen of the Philippines.


He is a nephew of Imelda Marcos

1980 Constructed residential house in Brgy Malbog,


Talosa, Leyte to establish legal residence therein. Later he
served as Brgy captain in such place.

1984 Batasan Election and in 1986 "snap" Presidential


Election Acted as the Campaign Manager of the Kilusang
Bagong Lipunan (KBL) in Leyte where he voted

After EDSA Peoples Power Revolution of 1986 He and his


immediate family left PH and sought asylum in USA which
USA granted. While abroad, he took special studies on the
development of Leyte-Samar and international business
finance

Early part of 1987 He attempted to come back to the


Philippines to run for a congressional seat in Leyte. By
March of the same year, his family booked a flight back but
it was somehow aborted

September 1991 He received a letter from U.S.


Immigration and Naturalization Service, informing him that
he should depart from the USA at his expense on or before
23 August 1992, if not, it may result in the withdrawal of
voluntary departure and action being taken to deportation.
Upon receipt, he departed from the U.S. for the Philippines,
arriving on December 1991 apparently without any
government document.

February 1992 He registered himself anew in the same


Brgy

Days after that, Advincula filed a petition to exclude him


from the list of voters alleging that he is a resident of
Massachusetts USA, his occupation is in the USA, and
having just arrived in PH, he doesnt have the required 1year residence in PH and 6-month residence in Talosa to
qualify him as voter

February 1992 Romualdez answer contended that he has


been a resident since the early 1980's, and that he has not
abandoned residence by his physical absence therefrom
during the period from 1986 up to the third week of
December 1991

MTC favored Romualdez. It was however reversed by the


RTC.

ISSUE: WON Romualdez voluntarily left the country and thereby


abandoned his residence in Malbog, Tolosa, Leyte.
HOLDING: NO!
In election cases, the Court treats domicile and residence as
synonymous terms which imports not only an intention to
reside in a fixed place but also personal presence in that
place, coupled with conduct indicative of such intention.

CO N F L I C T O F L AW S
N a t i o n a l i t y & D o m i c i l i a r y T h e o r y | 35
"Domicile" denotes a fixed permanent residence to which when
absent for business or pleasure, or for like reasons, one intends
to return. That residence, in Romualdez case, was established
during the early 1980's to be at Barangay Malbog, Tolosa, Leyte.
Residence thus acquired, however, may be lost by
adopting another choice of domicile. In order, in turn, to
acquire a new domicile by choice, there must concur:
(1) residence or bodily presence in the new locality;

Imelda amended her COC, changing the entry seven months


to since childhood in item no. 8. The provincial election
supervisor refused to admit the amended COC for the reason
that it was filed out of time. Imelda, thus, filed her amended
COC with Comelec's head office in Manila.
Petitioners Answer

Imelda averred that the entry of the word "seven" in


her original Certificate of Candidacy was the result of
an "honest misinterpretation" which she sought to
rectify by adding the words "since childhood" in her
Amended/Corrected Certificate of Candidacy and that
"she has always maintained Tacloban City as her
domicile or residence.

In an accompanying affidavit, she stated that her


domicile is Tacloban City, a component of the First
District, to which she always intended to return
whenever absent and which she has never
abandoned.

In her memorandum, she tried to discredit petitioner's


theory of disqualification by alleging that she has been
a resident of the First Legislative District of Leyte since
childhood, although she only became a resident of the
Municipality of Tolosa for seven months. She asserts
that she has always been a resident of Tacloban City, a
component of the First District, before coming to the
Municipality of Tolosa.

(2) an intention to remain there (animus manend) for


an indefinite period of time, and;
(3) an intention to abandon the old domicile (animus
non revertendi).
Further, the change of residence must be voluntary; and
the residence at the place chosen for the new domicile must be
actual.
The political situation brought about by the "People's Power
Revolution" must have truly caused great apprehension to the
Romualdezes, as well as a serious concern over the safety and
welfare of the members of their families. Their going into selfexile until conditions favorable to them would have somehow
stabilized is understandable. Certainly, their sudden
departure from the country cannot be described as
"voluntary," or as "abandonment of residence" at least in
the context that these terms are used in applying the
concept of "domicile by choice."
G.R. No. 119976 September 18, 1995
IMELDA
ROMUALDEZ-MARCOS, petitioner,
vs.
COMMISSION
ON
ELECTIONS
and
CIRILO
ROY
MONTEJO, respondents.
FACTS: On March 8, 1995, Petitioner Imelda Romualdez-Marcos
filed her Certificate of Candidacy for the position of
Representative of the First District of Leyte, stating that she is 7months resident in the said district. She provided the following
information in item no. 8:

On April 24, 1995, COMELEC Second Division came up with a


resolution
1.
2.
3.

finding private respondent's Petition for Disqualification


meritorious;
striking off Imeldas Corrected/Amended Certificate of
Candidacy of March 31, 1995; and
canceling her original Certificate of Candidacy

COMELEC SECOND DIVISION RULING:

Private respondent Cirilo Roy Montejo, the incumbent


Representative of the First District of Leyte and a candidate for
the same position, filed a "Petition for Cancellation and
Disqualification" with the Commission on Elections alleging that
Imelda did not meet the constitutional requirement for
residency.

The Comelec in division ruled that Imeldas defense of honest


mistake or misrepresentation is devoid of merit and that she
did not comply with the 1 year residency requirement of the
Constitution. The COmelec in division found that when Imelda
return to the Philippines in 1991, the residence she chose was
not Tacloban but San Juan, Metro Manila. Thus, her animus
revertendi is pointed to Metro Manila and not Tacloban.
Moreover when Imelda chose to stay in Ilocos and later on in
Manila, coupled with her intention to stay there by registering as
a voter there and expressly declaring that she is a resident of
that place, she is deemed to have abandoned Tacloban City,
where she spent her childhood and school days, as her place of
domicile.

Private respondents contention

The Comelec en banc affirmed this ruling.

RESIDENCE IN THE CONSTITUENCY WHERE I SEEK TO


BE ELECTED IMMEDIATELY PRECEDING THE ELECTION:
__________ Years and seven Months.

Imelda lacked the Constitution's one year residency


requirement for candidates for the House of
Representatives on the evidence of declarations made
by her in Voter Registration Record and in her
Certificate of Candidacy.
Imelda is not a resident of Tacloban City (a component
of the First District of Leyte) but of Barangay Olot,
Tolosa, Leyte.

ISSUE: Whether or not petitioner was a resident, for


election purposes, of the First District of Leyte for a
period of one year at the time of the May 9, 1995
elections.
HELD: YES

CO N F L I C T O F L AW S
N a t i o n a l i t y & D o m i c i l i a r y T h e o r y | 36
Principles
While the COMELEC seems to be in agreement with the general
proposition that for the purposes of election law, residence is
synonymous with domicile, the Resolution reveals a tendency to
substitute or mistake the concept of domicile for actual
residence, a conception not intended for the purpose of
determining a candidate's qualifications for election to the
House of Representatives as required by the 1987 Constitution.
As it were, residence, for the purpose of meeting the
qualification for an elective position, has a settled meaning in
our jurisdiction.
Article 50 of the Civil Code decrees that "[f]or the exercise of
civil rights and the fulfillment of civil obligations, the domicile of
natural persons is their place of habitual residence." In Ong
vs. Republic 20 this court took the concept of domicile to mean
an individual's "permanent home", "a place to which, whenever
absent for business or for pleasure, one intends to return, and
depends on facts and circumstances in the sense that they
disclose intent." 21 Based on the foregoing, domicile includes the
twin elements of "the fact of residing or physical presence in a
fixed place" and animus manendi, or the intention of returning
there permanently.
Residence, in its ordinary conception, implies the factual
relationship of an individual to a certain place. It is the physical
presence of a person in a given area, community or country. The
essential distinction between residence and domicile in law is
that residence involves the intent to leave when the purpose for
which the resident has taken up his abode ends. One may seek
a place for purposes such as pleasure, business, or health. If a
person's intent be to remain, it becomes his domicile; if his
intent is to leave as soon as his purpose is established it is
residence.22 It is thus, quite perfectly normal for an individual to
have different residences in various places. However, a person
can only have a single domicile, unless, for various reasons, he
successfully abandons his domicile in favor of another domicile
of choice. In Uytengsu vs. Republic, 23 we laid this distinction
quite clearly:
There is a difference between domicile and residence.
"Residence" is used to indicate a place of abode,
whether permanent or temporary; "domicile" denotes
a fixed permanent residence to which, when absent,
one has the intention of returning. A man may have a
residence in one place and a domicile in another.
Residence is not domicile, but domicile is residence
coupled with the intention to remain for an unlimited
time. A man can have but one domicile for the same
purpose at any time, but he may have numerous places
of residence. His place of residence is generally his
place of domicile, but it is not by any means
necessarily so since no length of residence without
intention of remaining will constitute domicile.
For political purposes the concepts of residence and
domicile are dictated by the peculiar criteria of political
laws. As these concepts have evolved in our election law,
what has clearly and unequivocally emerged is the fact
that
residence
for
election
purposes
is
used
synonymously with domicile.

In Co
vs. Electoral
Tribunal
of
the
House
of
Representatives, 31 this Court concluded that the framers of the
1987 Constitution obviously adhered to the definition given to
the term residence in election law, regarding it as having the
same meaning as domicile. 32
Has petitioner Imelda Romualdez Marcos satisfied the
residency requirement mandated by Article VI, Sec. 6 of
the 1987 Constitution?
ON HONEST MISTAKE
It stands to reason therefore, that petitioner merely committed
an honest mistake in jotting the word "seven" in the space
provided for the residency qualification requirement. The
circumstances leading to her filing the questioned entry
obviously resulted in the subsequent confusion which prompted
petitioner to write down the period of her actual stay in Tolosa,
Leyte instead of her period of residence in the First district,
which was "since childhood" in the space provided. These
circumstances and events are amply detailed in the COMELEC's
Second Division's questioned resolution, albeit with a different
interpretation. For instance, when herein petitioner announced
that she would be registering in Tacloban City to make her
eligible to run in the First District, private respondent Montejo
opposed the same, claiming that petitioner was a resident of
Tolosa, not Tacloban City. Petitioner then registered in her place
of actual residence in the First District, which is Tolosa, Leyte, a
fact which she subsequently noted down in her Certificate of
Candidacy. A close look at said certificate would reveal the
possible source of the confusion: the entry for residence (Item
No. 7) is followed immediately by the entry for residence in the
constituency where a candidate seeks election thus:
7. RESIDENCE (complete Address): Brgy. Olot, Tolosa,
Leyte
POST
OFFICE
ADDRESS
FOR
PURPOSES: Brgy. Olot, Tolosa, Leyte

ELECTION

8. RESIDENCE IN THE CONSTITUENCY WHERE I SEEK TO


BE
ELECTED
IMMEDIATELY
PRECEDING
THE
ELECTION:_________ Years and Seven Months.
Having been forced by private respondent to register in her
place of actual residence in Leyte instead of petitioner's claimed
domicile, it appears that petitioner had jotted down her period
of stay in her legal residence or domicile. The juxtaposition of
entries in Item 7 and Item 8 the first requiring actual
residence and the second requiring domicile coupled with the
circumstances surrounding petitioner's registration as a voter in
Tolosa obviously led to her writing down an unintended entry for
which she could be disqualified. This honest mistake should not,
however, be allowed to negate the fact of residence in the First
District if such fact were established by means more convincing
than a mere entry on a piece of paper.
ON PETITIONERS DOMICILE
We have stated, many times in the past, that an individual does
not lose his domicile even if he has lived and maintained

CO N F L I C T O F L AW S
N a t i o n a l i t y & D o m i c i l i a r y T h e o r y | 37
residences in different places. Residence, it bears repeating,
implies a factual relationship to a given place for various
purposes. The absence from legal residence or domicile to
pursue a profession, to study or to do other things of a
temporary or semi-permanent nature does not constitute loss of
residence. Thus, the assertion by the COMELEC that "she could
not have been a resident of Tacloban City since childhood up to
the time she filed her certificate of candidacy because she
became a resident of many places" flies in the face of settled
jurisprudence in which this Court carefully made distinctions
between (actual) residence and domicile for election law
purposes. In Larena vs. Teves, 33 supra, we stressed:
[T]his court is of the opinion and so holds that a person
who has his own house wherein he lives with his family
in a municipality without having ever had the intention
of abandoning it, and without having lived either alone
or with his family in another municipality, has his
residence in the former municipality, notwithstanding
his having registered as an elector in the other
municipality in question and having been a candidate
for various insular and provincial positions, stating
every time that he is a resident of the latter
municipality.
More significantly, in Faypon vs. Quirino,

34

We explained that:

A citizen may leave the place of his birth to look for


"greener pastures," as the saying goes, to improve his
lot, and that, of course includes study in other places,
practice of his avocation, or engaging in business.
When an election is to be held, the citizen who left his
birthplace to improve his lot may desire to return to his
native town to cast his ballot but for professional or
business reasons, or for any other reason, he may not
absent himself from his professional or business
activities; so there he registers himself as voter as he
has the qualifications to be one and is not willing to
give up or lose the opportunity to choose the officials
who are to run the government especially in national
elections. Despite such registration, the animus
revertendi to his home, to his domicile or residence of
origin has not forsaken him. This may be the
explanation why the registration of a voter in a place
other than his residence of origin has not been deemed
sufficient to constitute abandonment or loss of such
residence. It finds justification in the natural desire and
longing of every person to return to his place of birth.
This strong feeling of attachment to the place of one's
birth must be overcome by positive proof of
abandonment for another.
From the foregoing, it can be concluded that in its above-cited
statements supporting its proposition that petitioner was
ineligible to run for the position of Representative of the First
District of Leyte, the COMELEC was obviously referring to
petitioner's various places of (actual) residence, not her
domicile. In doing so, it not only ignored settled jurisprudence
on residence in election law and the deliberations of the
constitutional commission but also the provisions of the
Omnibus Election Code (B.P. 881). 35

SET OF FACTS WHICH ESTABLISH THE FACT OF PETITIONER'S


DOMICILE

While petitioner was born in Manila, as a minor she


naturally followed the domicile of her parents. She grew
up in Tacloban, reached her adulthood there and
eventually established residence in different parts of
the country for various reasons.

Even during her husband's presidency, at the height of


the Marcos Regime's powers, petitioner kept her close
ties to her domicile of origin by establishing residences
in Tacloban, celebrating her birthdays and other
important personal milestones in her home province,
instituting well-publicized projects for the benefit of her
province and hometown, and establishing a political
power base where her siblings and close relatives held
positions of power either through the ballot or by
appointment, always with either her influence or
consent. These well-publicized ties to her domicile of
origin are part of the history and lore of the quarter
century of Marcos power in our country.
TACLOBAN WAS PETITIONER'S DOMICILE OF ORIGIN
First, minor follows the domicile of his parents.
As domicile, once acquired is retained until a new one is gained,
it follows that in spite of the fact of petitioner's being born in
Manila, Tacloban, Leyte was her domicile of origin by operation
of law. This domicile was not established only when her father
brought his family back to Leyte contrary to private
respondent's averments.
Second, domicile of origin is not easily lost.
To successfully
demonstrate:

effect

change

of

domicile,

one

must

1. An actual removal or an actual change of domicile;


2. A bona fide intention of abandoning the former place
of residence and establishing a new one; and
3. Acts which correspond with the purpose.
In the absence of clear and positive proof based on these
criteria, the residence of origin should be deemed to continue.
Only with evidence showing concurrence of all three
requirements can the presumption of continuity or residence be
rebutted, for a change of residence requires an actual and
deliberate abandonment, and one cannot have two legal
residences at the same time. 38 In the case at bench, the
evidence adduced by private respondent plainly lacks the
degree of persuasiveness required to convince this court that an
abandonment of domicile of origin in favor of a domicile of
choice indeed occurred. To effect an abandonment requires the
voluntary act of relinquishing petitioner's former domicile with
an intent to supplant the former domicile with one of her own
choosing (domicilium voluntarium).

CO N F L I C T O F L AW S
N a t i o n a l i t y & D o m i c i l i a r y T h e o r y | 38
In this connection, it cannot be correctly argued that petitioner
lost her domicile of origin by operation of law as a result of her
marriage to the late President Ferdinand E. Marcos in 1952. For
there is a clearly established distinction between the Civil Code
concepts of "domicile" and "residence." 39 The presumption that
the wife automatically gains the husband's domicile by
operation of law upon marriage cannot be inferred from the use
of the term "residence" in Article 110 of the Civil Code because
the Civil Code is one area where the two concepts are well
delineated. Dr. Arturo Tolentino, writing on this specific area
explains:
In the Civil Code, there is an obvious difference
between domicile and residence. Both terms imply
relations between a person and a place; but in
residence, the relation is one of fact while in domicile it
is legal or juridical, independent of the necessity of
physical presence. 40
Article 110 of the Civil Code provides:
Art. 110. The husband shall fix the residence of the
family. But the court may exempt the wife from living
with the husband if he should live abroad unless in the
service of the Republic.
A survey of jurisprudence relating to Article 110 or to the
concepts of domicile or residence as they affect the female
spouse upon marriage yields nothing which would suggest that
the female spouse automatically loses her domicile of origin in
favor of the husband's choice of residence upon marriage.
Article 110 is a virtual restatement of Article 58 of the Spanish
Civil Code of 1889 which states:

difference could, for the sake of family unity, be reconciled only


by allowing the husband to fix a single place of actual residence.
Very significantly, Article 110 of the Civil Code is found under
Title V under the heading: RIGHTS AND OBLIGATIONS BETWEEN
HUSBAND AND WIFE. Immediately preceding Article 110 is
Article 109 which obliges the husband and wife to live together,
thus:
Art. 109. The husband and wife are obligated to live
together, observe mutual respect and fidelity and
render mutual help and support.
The duty to live together can only be fulfilled if the husband and
wife are physically together. This takes into account the
situations where the couple has many residences (as in the case
of the petitioner). If the husband has to stay in or transfer to any
one of their residences, the wife should necessarily be with him
in order that they may "live together." Hence, it is illogical to
conclude that Art. 110 refers to "domicile" and not to
"residence." Otherwise, we shall be faced with a situation where
the wife is left in the domicile while the husband, for
professional or other reasons, stays in one of their (various)
residences. As Dr. Tolentino further explains:
Residence and Domicile Whether the word
"residence" as used with reference to particular
matters is synonymous with "domicile" is a question of
some difficulty, and the ultimate decision must be
made from a consideration of the purpose and intent
with which the word is used. Sometimes they are used
synonymously, at other times they are distinguished
from one another.
xxx xxx xxx

La mujer esta obligada a seguir a su marido donde


quiera que fije su residencia. Los Tribunales, sin
embargo, podran con justa causa eximirla de esta
obligacion cuando el marido transende su residencia a
ultramar o' a pais extranjero.
Note the use of the phrase "donde quiera su fije de residencia"
in the aforequoted article, which means wherever (the
husband) wishes to establish residence. This part of the article
clearly contemplates only actual residence because it refers to a
positive act of fixing a family home or residence. Moreover, this
interpretation is further strengthened by the phrase "cuando el
marido translade su residencia" in the same provision which
means, "when the husband shall transfer his residence,"
referring to another positive act of relocating the family to
another home or place of actual residence. The article obviously
cannot be understood to refer to domicile which is a fixed,
fairly-permanent concept when it plainly connotes the possibility
of transferring from one place to another not only once, but as
often as the husband may deem fit to move his family, a
circumstance more consistent with the concept of actual
residence.
The right of the husband to fix the actual residence is in
harmony with the intention of the law to strengthen and unify
the family, recognizing the fact that the husband and the wife
bring into the marriage different domiciles (of origin). This

Residence in the civil law is a material fact, referring to


the physical presence of a person in a place. A person
can have two or more residences, such as a country
residence and a city residence. Residence is acquired
by living in place; on the other hand, domicile can exist
without actually living in the place. The important thing
for domicile is that, once residence has been
established in one place, there be an intention to stay
there permanently, even if residence is also established
in
some
other
place. 41
In fact, even the matter of a common residence between the
husband and the wife during the marriage is not an iron-clad
principle; In cases applying the Civil Code on the question of a
common matrimonial residence, our jurisprudence has
recognized certain situations 42 where the spouses could not be
compelled to live with each other such that the wife is either
allowed to maintain a residence different from that of her
husband or, for obviously practical reasons, revert to her original
domicile (apart from being allowed to opt for a new one). In De
la Vina vs. Villareal 43 this Court held that "[a] married woman
may acquire a residence or domicile separate from that of her
husband during the existence of the marriage where the
husband has given cause for divorce." 44 Note that the Court
allowed the wife either to obtain new residence or to choose a

CO N F L I C T O F L AW S
N a t i o n a l i t y & D o m i c i l i a r y T h e o r y | 39
new domicile in such an event. In instances where the wife
actually opts, .under the Civil Code, to live separately from her
husband either by taking new residence or reverting to her
domicile of origin, the Court has held that the wife could not be
compelled to live with her husband on pain of contempt.
Parenthetically when Petitioner was married to then
Congressman Marcos, in 1954, petitioner was obliged by
virtue of Article 110 of the Civil Code to follow her husband's
actual place of residence fixed by him. The problem here is that
at that time, Mr. Marcos had several places of residence, among
which were San Juan, Rizal and Batac, Ilocos Norte. There is no
showing which of these places Mr. Marcos did fix as his family's
residence. But assuming that Mr. Marcos had fixed any of these
places as the conjugal residence, what petitioner gained upon
marriage was actual residence. She did not lose her domicile of
origin.
On the other hand, the common law concept of "matrimonial
domicile" appears to have been incorporated, as a result of our
jurisprudential experiences after the drafting of the Civil Code of
1950, into the New Family Code. To underscore the difference
between the intentions of the Civil Code and the Family Code
drafters, the term residence has been supplanted by the term
domicile in an entirely new provision (Art. 69) distinctly different
in meaning and spirit from that found in Article 110. The
provision recognizes revolutionary changes in the concept of
women's rights in the intervening years by making the choice of
domicile a product of mutual agreement between the
spouses. 46
Without as much belaboring the point, the term residence may
mean one thing in civil law (or under the Civil Code) and quite
another thing in political law. What stands clear is that insofar as
the Civil Code is concerned-affecting the rights and obligations
of husband and wife the term residence should only be
interpreted to mean "actual residence." The inescapable

conclusion derived from this unambiguous civil law delineation


therefore, is that when petitioner married the former President
in 1954, she kept her domicile of origin and merely gained a
new home, not a domicilium necessarium.
Even assuming for the sake of argument that petitioner gained a
new "domicile" after her marriage and only acquired a right to
choose a new one after her husband died, petitioner's acts
following her return to the country clearly indicate that she not
only impliedly but expressly chose her domicile of origin
(assuming this was lost by operation of law) as her domicile.
This "choice" was unequivocally expressed in her letters to the
Chairman of the PCGG when petitioner sought the PCGG's
permission to "rehabilitate (our) ancestral house in Tacloban and
Farm in Olot, Leyte. . . to make them livable for the Marcos
family to have a home in our homeland." 47 Furthermore,
petitioner obtained her residence certificate in 1992 in Tacloban,
Leyte, while living in her brother's house, an act which supports
the domiciliary intention clearly manifested in her letters to the
PCGG Chairman. She could not have gone straight to her home
in San Juan, as it was in a state of disrepair, having been
previously looted by vandals. Her "homes" and "residences"
following her arrival in various parts of Metro Manila merely
qualified as temporary or "actual residences," not domicile.
Moreover, and proceeding from our discussion pointing out
specific situations where the female spouse either reverts to her
domicile of origin or chooses a new one during the subsistence
of the marriage, it would be highly illogical for us to assume that
she cannot regain her original domicile upon the death of her
husband absent a positive act of selecting a new one where
situations exist within the subsistence of the marriage itself
where the wife gains a domicile different from her husband.
In the light of all the principles relating to residence and
domicile enunciated by this court up to this point, we are
persuaded that the facts established by the parties weigh
heavily in favor of a conclusion supporting petitioner's claim of
legal residence or domicile in the First District of Leyte.

S-ar putea să vă placă și